Therapy Flashcards

1
Q

Task 64
A 21-year-old patient called a doctor to the house urgently due to complaints of a large amount of scarlet blood from the rectum during bowel movements , frequent stools up to 5 times a day, an increase in body temperature to 38 ° C, general weakness.
The appearance of blood in the stool was noted during the last 2 months , the stool gradually began to become more frequent, the consistency changed to mushy, there was general weakness, low-grade fever, deterioration within 3 days.
On examination, the skin is pale.
BP - 110/70 mm Hg. Art. Pulse - 96 per minute, rhythmic. The tongue is coated with a white coating.
The abdomen is soft, painful in the left iliac region . Symptoms of peritoneal irritation are negative.
The patient was sent to a hospital for the selection of therapy and further examination in order to clarify the diagnosis.
The task
1. Formulate a preliminary diagnosis.
2. With what diseases is it necessary to carry out a differential diagnosis?
3. What research needs to be done for this patient?
4. What groups of drugs can be used to treat this patient?
5. What are the possible complications of this disease?

A

1:Diagnosis -Ulcerative colitis, proctosigmoiditis, moderate attack.
2: differential diagnosis includes:ischemic colitis ,acute dysentery , Crohn’s disease, pseudomembranous colitis,hemorrhoids
3:research needs; General blood test, b / x blood test (glucose, cholesterol, lipids, K, Na, Ca, protein and fractions, (anemia:decreased red blood cells ,CRP, alkaline phosphatase, GGTP, bilirubin, amylase, creatinine),
* coprogram, general urine analysis, fecal calprotectin, ANCA, ASCA, toxin A / B,
* calcium;
*Fecal calprotectin (↑100 mcg/g).
stool culture for dysbiosis , RNGA with pseudotuberculosis antigen , yersiniosis antigen
* ultrasound of the abdominal cavity, EFGDS, fibrocolonoscopy + intestinal biopsy.
*X-ray
4: treatment; 1. Depending on the degree of inflammation . 5-Ask (mesalazine) preparations, - 0.5-1.0 g tablets and local therapy proctitis(mesalazine in candles or
Left sided colitis (Mesalazine in enemas (salofalk 2 grams 1-2 times a day
For moderate course we prescribe prednisone 0.005 initially 40-60 mg a day 10 to 14 days improvement before gradual reduction of 10mg of prednisone
( Prednisolone, Budesonide ) in enemas (budesonide )systemic and local - rectally, for complicated course of the disease we use cytostatics (Metatrexate 25mg + folic acid to prevent side effects , Azathioprine 2-3mg per kg of patients weight ).
Genetically engineered biological therapy. Symptomatic: antispasmodics, intestinal antiseptics, probiotics.
Bowel perforation, sepsis,
** Additional information

  1. The preliminary diagnosis for this patient is acute lower gastrointestinal bleeding, likely caused by inflammatory bowel disease (IBD), such as ulcerative colitis or Crohn’s disease, given the symptoms of rectal bleeding, frequent stools, and abdominal pain.
  2. Differential diagnosis for this clinical case includes:
    - Infectious colitis
    - Ischemic colitis
    - Diverticulitis
    - Colorectal cancer
    - Hemorrhoids
    - Anal fissures
  3. Research that needs to be done for this patient includes a colonoscopy or sigmoidoscopy to visualize the colon and rectum and identify the source of bleeding, as well as laboratory tests to assess the patient’s blood count and inflammatory markers.
  4. Drugs that can be used to treat this patient include anti-inflammatory medications, such as corticosteroids and mesalamine, to reduce inflammation in the colon and rectum, as well as antibiotics if there is evidence of infection. Iron supplements may also be necessary to address anemia caused by bleeding.
  5. Possible complications of IBD include severe bleeding, bowel obstruction, and the formation of abscesses or fistulas. In some cases, IBD can also increase the risk of developing colorectal cancer.
How well did you know this?
1
Not at all
2
3
4
5
Perfectly
2
Q

Task 65
Patient K., 29 years old, consulted a local general practitioner with complaints of frequent weak aching pains in the epigastric region , which decrease after eating; feeling of quick satiety; sour belching. Also notes general weakness, instability of the stool.
These complaints periodically bother over the past 5 years, the current deterioration in health - 1.5 weeks ago.
He took antacids on his own with a short-term positive effect .
On objective examination: the condition is satisfactory. Skin and visible mucous membranes of normal color, clean. On the part of the cardiovascular, respiratory systems without features. The tongue is moist, coated with a white coating. The abdomen is of normal shape, soft on palpation, painful in the epigastric region.
In the clinical analysis of blood: erythrocytes - 4.8 × 10 12 / l, hemoglobin - 140 g / l, CP - 0.87, platelets - 380 × 10 9 / l, leukocytes - 7.2 × 10 9 / l, ESR - 16 mm / h.
In a biochemical blood test: ALT - 21 U / L, AST - 18 U / L, amylase - 53 U / L, alkaline phosphatase
- 78 U / l.
FGDS performed: there is a large amount of mucus in the lumen of the stomach. Phenomena of moderate atrophy in the pyloric region of the stomach, edema and hyperemia in the fundus of the stomach . Active peristalsis. The stomach expands well with air.
H.pylori test +++.
The task
1. Formulate the most likely diagnosis.
2. Is there a need for eradication therapy in this case?
3. Suggest a treatment plan.
4. What factors of natural protection of the gastric mucosa and factors of aggression do you know?
5. What complications of this disease are possible in the absence of specific treatment?

A

1: Diagnosis Chronic atrophic gastritis, HP-associated, exacerbation phase. Functional dyspepsia.
2: The presence of zones of atrophy in HP- associated gastritis is an indication for mandatory eradication therapy, since this is a preventive measure to prevent precancerous changes in the gastric mucosa and cancer itself.
3:treatment: Sparing diet, table number 1.
-Triple regimen of eradication therapy, which includes PPIs-rabeprazole 20mg (in a standard dose 2 times a day) + Clarithromycin (500 mg 2 times a day) + Amoxicillin (1000 mg 2 times a day) for 10-14 days
Or
- A four-component regimen based on Bismuth tripotassium dicitrate (120 mg 4 times a day) in combination with PPIs (in a standard dose 2 times a day), Tetracycline (500 mg 4 times a day), Metronidazole (500 mg 3 times a day) within 10 days ;
- prokinetics (Domperidone 10mg orally 3-4 times a day ) to eliminate the symptoms of functional dyspepsia; probiotics
4: A) Factors of aggression : hydrochloric acid, pepsin, violation of the evacuation function of the stomach, duodeno-gastric reflux, HP infection;
B) factors of protection of the mucous membrane : mucus formation, secretion of bicarbonates, proper blood flow, regeneration of the epithelium, prostaglandins, immune defense.
5 complications:Persistence of HP infection can lead to the formation of mucosal ulcers, complicated by bleeding, perforation, penetration into neighboring organs, the formation of cicatricial deformities, and malignancy.
With atrophy of the gastric mucosa due to HP infection, it can be replaced by intestinal epithelium (metaplasia), and then epithelial neoplasia.
Bleeding
***
Additional information

  1. The most likely diagnosis for this patient is chronic gastritis, likely caused by H. pylori infection, given the symptoms of epigastric pain, sour belching, and atrophy in the pyloric region of the stomach seen on FGDS.
  2. Yes, there is a need for eradication therapy in this case as the H. pylori test was positive. H. pylori infection is a common cause of chronic gastritis and can lead to complications such as peptic ulcers and gastric cancer if left untreated.
  3. The treatment plan for this patient should include a course of antibiotics to eradicate the H. pylori infection, along with proton pump inhibitors to reduce gastric acid secretion and promote healing of the gastric mucosa. The patient should also be advised to avoid alcohol, spicy foods, and other irritants that can exacerbate gastritis symptoms.
  4. Factors of natural protection of the gastric mucosa include the mucus layer, bicarbonate secretion, and blood flow to the gastric mucosa. Factors of aggression include H. pylori infection, NSAID use, alcohol consumption, and stress.
  5. Possible complications of chronic gastritis in the absence of specific treatment include peptic ulcers, gastric bleeding, and gastric cancer. Eradication of H. pylori infection can help prevent these complications and improve the patient’s overall prognosis.
How well did you know this?
1
Not at all
2
3
4
5
Perfectly
3
Q

Task 41
Patient K., 50 years old, turned to the local doctor in connection with the first attack of heartbeat, accompanied by muscle tremors, weakness, slight difficulty in breathing . The attack occurred about 2 hours ago with severe emotional stress. Previously, with regular medical examination, no diseases were detected, blood pressure was always within normal limits. Previously taken ECG showed no pathological changes. She tolerates very significant physical activity well.
On examination: clear consciousness. The skin is of normal color and moisture. In the lungs, vesicular breathing, RR - 18 per minute. The boundaries of relative cardiac dullness are within normal limits. Arrhythmic heart sounds, no murmurs, heart rate - 144 beats per minute, pulse - 108 per minute. BP - 130/80 mm Hg. Art. The liver is not enlarged.
There are no peripheral edema. Body temperature 36.9 ° C.
Presented ECG lead II (speed 25 mm / s):

The task
1. Suggest the most likely diagnosis.
2. What are the deviations from the norm, visible on the presented ECG, and formulate an ECG-conclusion.
3. What syndrome is the leading one in the clinical picture of this disease?
4. Among what similar conditions are differential diagnostics required?
5. With the introduction of what drugs should you begin to relieve this emergency?

A

1:Diagnosis : Atrial fibrillation (Idiopathic paroxysmal atrial fibrillation, tachysystole, hemodynamically insignificant par-
oxysm.) Stage 2A
2: The axis is deflected to the left, (looking at Lead 1 and AVF = lead 1 QRS complex is positive and negative in avF
-the rhythm is non-sinus
- atrial fibrillation (there is no p wave ,
-different rr intervals,
-there are f waves).
Paroxysmal form of FP, unclear etiology.
3:The leading syndrome is rhythm disturbances .
4:differential diagnosis: - among FPs by etiology; with other tachyarrhythmias (for example, atrial flutter , mb with a stretch sinus tachycardia, paroxysmal supraventricular tachycardia - but there the rhythm is correct and there are no f waves)
5: The attack lasts less than 48 hours, we restore the rhythm - we inject heparin 5000 iu and an antiarrhythmic (if there is no organic damage to the heart - propafenone, procainamide). Our patient can use propafenone, 300-600 mg.
*Edoxaban 60mg once daily
*You can amiodarone 300-400 mg drip in glucose 500 dangerous by the development of blockages).
If it was more than 48 hours, they would give AK (Applied kinesiology (AK) for 3 weeks, and then restore the rhythm (HP / electro), and then again AK therapy with the possibility of long-term use (if there is a risk of TE)
* Next, the patient is referred to a local therapist, it is necessary to calculate the risk of thromboembolic complications and bleeding in order to resolve the issue of prescribing anticoagulants .
This patient does not need, does not score 2 points for the appointment of AK. Optional: if persistent or persistent FFP
Heart rate control is very important (avoid tachysystole): beta-blockers or nondihydropyridine ACCs (verapamil, diltiazem).
Very rare - amiodarone.
We also calculate the risk of thromboembolic complications and, if necessary, AK therapy. 1. CHERNOGORIUK
Paroxysmal atrial fibrillation
1. The most likely diagnosis for this patient is paroxysmal supraventricular tachycardia (PSVT), given the symptoms of sudden onset of palpitations, muscle tremors, and weakness, as well as the arrhythmic heart sounds and rapid heart rate seen on examination.
2. The ECG shows a narrow QRS complex tachycardia with a regular rhythm and a heart rate of approximately 144 beats per minute. There are no visible P waves, indicating that the tachycardia is likely originating from a site above the ventricles, such as the atria or the AV node. The ECG conclusion would be “Narrow complex tachycardia consistent with PSVT.”
3. The leading syndrome in the clinical picture of PSVT is palpitations, which can be accompanied by other symptoms such as weakness, dizziness, and shortness of breath.
4. Differential diagnosis for PSVT includes other types of supraventricular tachycardia, such as atrial fibrillation or atrial flutter, as well as ventricular tachycardia.
5. The introduction of drugs to relieve this emergency should begin with vagal maneuvers, such as the Valsalva maneuver or carotid sinus massage, to try to slow the heart rate. If these maneuvers are not effective, adenosine can be administered intravenously to interrupt the reentry circuit responsible for PSVT. Calcium channel blockers or beta-blockers may also be used to prevent future episodes of PSVT. In severe cases, electrical cardioversion may be necessary to restore normal sinus rhythm.

How well did you know this?
1
Not at all
2
3
4
5
Perfectly
4
Q

Task 42
A 42-year-old man has come to the clinic with an attack of intense constricting pain behind the sternum without clear radiation, accompanied by nausea, profuse sweating, and dyspnea. By the time the patient came to the clinic, the attack had been continuing for about 3 hours. The patient has been smoking for many years, 10-15 cigarettes a day.
There is no history of arterial hypertension or other cardiovascular diseases, diabetes, neurological disorders, head trauma, coagulopathy and severe bleeding.
The patient’s consciousness is clear. The skin is pale, hyperhidrosis. Respiration is vesicular, without rale. Respiration rate is 18 brpm. Cardiac sounds are rhythmic. Heart rate is 90 bpm, arterial pressure is 130/80 mmHg. The abdomen is soft and painless in all regions. The liver is not enlarged.
ECG was urgently taken.

Task
1. Suppose the most likely diagnosis.
2. What abnormalities are visible on the given ECG? Present an ECG report.
3. What is the treatment plan of the general practitioner in this situation?
4. What medical emergencies should differential diagnosis be carried out with? 5. What laboratory markers should be used to confirm the diagnosis?

A

1.Diagnosis: Arteriosclerotic heart disease: acute transmural anterior extensive myocardial infarction,
acute phase. Killip class I based on the classification by T. Killip, J. Kimball.heart failure class 1(no evidence of heart failure)
2.ECG abnormalities include
*ST segment elevation is visible in leads v2,v3,v4,v5indicating acute myocardial infarction on the anterior heart
3.Treatment plan
* Emergency hospitalization. Before the arrival of the ambulance team:
###start drug therapy 1st step then consider revascularization
>oxygen
>morphine
>aspirin 325mg
>Nitrates
>Beta blockers
>Heparin
##consider clopidogrel 300-600mg or prasugrel 60mg if PCI is indicated
Our patient STEMI thus we should immediately transfer the patient to a cardiology facility and PCI is indicated within 90min and begin urgently angiography,then if coronary artery is amenable proceed to PCI I,and if it’s not amenable proceed to Emergency CABG(coronary artery bypass surgery)
##If PCI is not done within 90mins proceed to thrombolytic therapy with Atepelase ,if there is still persistent ischemia repeat thrombolytic therapy or do PCI if available but if no persistent ischemia you can do elective PCI within 24-72 hrs
###Post acute coronary syndrome do the following
>management of risk factors such as hypertension,diabetes,diet,sedentary lifestyle,obesity,smoking
>prescription of drugs like AcE inhibitors,clopidogrel 75mg once day ,maximum tolerated statin dose
* put the patient to bed, install a bedside cardiac monitor (if possible), anesthesia (Nitroglycerin 0.6mg sublingual - nitrospray or non-narcotic analgesic - ketorol 10 mg IM +morphine ) , chew aspirin 500 mg , clopidogrel 300 mg , heparin 5000 ME therapy - heparin PC. Ace inhibitors
4. Differential diagnosis
* Newly onset angina pectoris,
* acute pericarditis
* dissecting aortic aneurysm,
* pulmonary embolism
5. Laboratory markers of myocardial infarction
* Creatine-Kinase-MB isoform
* Cardiac Troponin T and I
** Additional information

  1. The most likely diagnosis for this patient is acute coronary syndrome (ACS), specifically an ST-elevation myocardial infarction (STEMI), given the symptoms of intense chest pain, nausea, sweating, and dyspnea, as well as the patient’s history of smoking.
  2. The ECG shows ST-segment elevation in leads II, III, and aVF, indicating an inferior wall STEMI. The ECG report would be “Inferior wall STEMI with ST-segment elevation in leads II, III, and aVF.”
  3. The treatment plan for the general practitioner in this situation should include immediate transfer of the patient to a hospital for urgent reperfusion therapy, such as percutaneous coronary intervention (PCI) or thrombolytic therapy. The patient should also be given aspirin and nitroglycerin to help relieve symptoms.
  4. Differential diagnosis for ACS includes unstable angina and non-ST elevation myocardial infarction (NSTEMI), as well as other conditions that can cause chest pain, such as pulmonary embolism or aortic dissection.
  5. Laboratory markers that can be used to confirm the diagnosis of ACS include cardiac troponin levels, which are typically elevated in patients with myocardial infarction. Other tests, such as a complete blood count and metabolic panel, may also be performed to assess the patient’s overall health status and help guide treatment decisions.
How well did you know this?
1
Not at all
2
3
4
5
Perfectly
5
Q

Task 44
A 69-year-old patient is sent to a clinic by a district doctor with complaints of severe dyspnea during slight physical activity, suffocation attacks at night. Deterioration of patient’s condition is observed about a week before hospitalization.
In history: 6 years ago the patient was diagnosed with dilated cardiomyopathy. The patient took the following selected medication: Hypothiazid (12.5 mg a day), Fosinoprilum (20 mg a day), and Caverdilolum (12.5 mg twice a day). There was weakness during physical activity.
The patient reported that cough appeared and body temperature increased to 37.4°С several days ago after hypothermia. Due to increasing weakness, the patient stopped taking Caverdilolum and drank plenty of liquid and took vitamins. The patient’s condition got worse.
The patient’s weight is 76 kg and the height is 168 cm. Cyanosis. At auscultation: stagnant rale from both sides in the lungs; cardiac sounds are dull, rhythmic, protodiastolic gallop rhythm. Heart rate is 105 bpm. Arterial pressure is 105/70 mmHg. There is symmetrical edema of lower extremities.
Blood tests showed sodium 138 mmol/L and potassium 3.9 mmol/L.
ECG was performed with the following results: sinus rhythm, left bundle branch block, ventricular extrasystoles, QRS 0.13 sec.
Echocardiography was performed with the following results: diffuse hypokinesia of the left ventricular walls, ejection fraction 36%, left ventricular end diastolic volume 69 mm.
Task
1. Suppose the most likely diagnosis.
2. What caused the severe condition of the patient? What contributed to the development of heart failure decompensation? Justify the diagnosis.
3. Make and justify a plan for additional examination of the patient.
4. What groups of medications should be administered to the patient? What decision should be made about the prescribed ACE inhibitor Fosinoprilum in connection with arterial pressure 105/70 mmHg (cancel, reduce or increase the dose)? What should be done about Carvedilol?
5. If, despite adequate therapy, it is impossible to achieve a stable compensation for heart failure, are there any other options for treatment?

A

1.Diagnosis: Dilated cardiomyopathy. Complete left bundle branch block. Rare ventricular extrasystole.
Chronic heart failure with reduced EF LVH (36%), stage II B, Functional Class IV.
2.what caused the severe conditions of this patient and justify the diagnosis
* According to the anamnesis, the diagnosis “dilated cardiomyopathy” was made to the patient 6 years ago (with ECHO-CG there is dilatation of the left ventricle, hypokinesia of the walls of the left ventricle). Currently, the severity of the condition is due to the phenomena of heart failure, the decompensation of which was facilitated by: viral infection, abundant fluid intake and drug withdrawal. The diagnosis of CHF and its stage are based on complaints (shortness of breath, dyspnea), objective data (cyanosis, edema of the lower extremities, wheezing in the lungs, protodiastolic gallop rhythm), the results of ECHO-KG (low left ventricular ejection fraction, left ventricular dilatation)
3.make and justify a plan for additional examination
* In addition to general clinical studies (general blood test, general urine analysis, biochemical blood test),kidney function test, liver function test (creatine level,BUN
* it is necessary to conduct an X-ray examination of the chest organs to exclude pneumonia, infiltrative lung diseases (the patient has an increase in body temperature, cough) in order to identify signs of cardiomegaly, signs of venous stasis in lungs, the presence of effusion in the pleural cavity;
* daily monitoring of ECG according to Holter in order to detect disturbances in the rhythm and conduction of the heart.
4.what group of medications should be administered to this patient and What decision should be made about the prescribed ACE inhibitor Fosinoprilum in connection with arterial pressure 105/70 mmHg (cancel, reduce or increase the dose)? What should be done about Carvedilol?
-The main groups of drugs for the treatment of this patient
* ACE inhibitors
* selective b-blockers
* diuretics (Hypothiazide, Furosemide)
* cardiac glycosides Digoxin (due to decompensation, large heart size and low left ventricular ejection fraction),
* antiplatelet agents (for the prevention of thromboembolic complications ).
### The ACE inhibitor fosinopril should be left to the patient in treatment, but the dose should be reduced due to low blood pressure. The ß-blocker Carvedilol should be re-prescribed to this patient .
5.despite adequate therapy it’s impossible to achieve a stable compensation for heart failure any other options for treatment
* If drug therapy is ineffective, electrophysiological and surgical methods of treatment are resorted to : two-chamber electrical stimulation of the heart using an implanted pacemaker (improves intracardiac hemodynamics, increases the systolic function of the ventricles, prevents the development of some complications); heart transplant. Indications for heart transplantation : rapid progression of CHF and lack of effect from the therapy; the occurrence of dangerous heart rhythm disturbances; high risk of thromboembolic complications.
** Additional information

  1. The most likely diagnosis for this patient is heart failure decompensation due to dilated cardiomyopathy, given the history of dilated cardiomyopathy and the symptoms of severe dyspnea, suffocation attacks, and edema of the lower extremities.
  2. The severe condition of the patient is likely caused by a worsening of the dilated cardiomyopathy, possibly triggered by the recent hypothermia and resulting in heart failure decompensation. The patient’s history of dilated cardiomyopathy, along with the presence of left bundle branch block, ventricular extrasystoles, and diffuse hypokinesia of the left ventricular walls seen on echocardiography, support this diagnosis.
  3. Additional examination of the patient should include laboratory tests to assess kidney function, electrolyte levels, and cardiac biomarkers, as well as imaging studies such as chest X-ray or CT scan to evaluate the extent of pulmonary congestion. A cardiac catheterization may also be considered to assess the degree of coronary artery disease and evaluate the patient’s candidacy for revascularization procedures.
  4. Medications that should be administered to the patient include diuretics to reduce fluid overload
How well did you know this?
1
Not at all
2
3
4
5
Perfectly
6
Q

Task 45
A 65-year-old patient, a pensioner, was admitted to the clinic with complaints of rare pulse, intermission, a feeling of heart fading and stopping, shortness of breath when climbing one flight of stairs, pressing pain behind the sternum during regular physical activity and brief loss of consciousness. The pain was relieved after 1-2 minutes after the patient took Nitroglycerin.
In history: the patient had a myocardial infarction 4 years ago. A year after, the patient developed anginal pain during normal physical activity. A week ago the patient felt intermissions, inspiratory dyspnea and noted brief loss of consciousness, which was the reason for hospitalization.

Objective clinical examination: the patient’s condition is of moderate severity, acrocyanosis, no edema. There is a small amount of non-sonorous fine bubbling rale in the lower parts of lungs. Cardiac sounds are muffled and non-rhythmic. Heart rate is 42 bpm. Pulse is 42 bpm. Arterial pressure is 110/65 mmHg.
The abdomen is soft and painless. The liver is 2 cm below the costal arch, its edge is smooth, rounded and slightly tender to palpation.
The ECG is recorded:
Task
1. Determine the syndromes and identify the main one.
2. Interpret the electrocardiogram.
3. Make a diagnosis.
4. Make a plan of additional examinations.
5. Prescribe treatment

A

1.Determine the syndromes and identify the main one
* Syndromes: rhythm and conduction disturbances, coronary insufficiency, chronic left ventricular failure.
Leading - a syndrome of rhythm and conduction disturbances .

2: interpretation of ECG
* Sinus rhythm, atrioventricular block II degree, Mobitz I (with Samoilov-Wenckebach periodicals)
3.Diagnosis
* Arteriosclerotic heart disease: angina pectoris, Functional Class III. Postinfarction cardio-
sclerosis.
Justification * angina pectoris because patient came in with pressing chest pain behind the sternum and it’s functional class III because patient losses consciousness,and he feels his heart fading and stopping with shortness of breath when climbing one flight of stairs , post MI sclerosis due to his anaemesis because patient had myocardial infarction for 4 years
4. Make a plan of additional examination
* General blood test,
* General urine test
* biochemical blood test (troponin T or I, glucose
* creatinine with the calculation of GFR, K, Na),
* dynamic ECG, coronary angiography
* echocardiography, chest X-ray.
*lipid profile
5. Prescribe treatment
* Urgent measures for an attack: put the patient to bed, provide fresh air, evaluate vital functions, ensure the removal of saliva
* Resuscitation measures (chest compressions and mechanical ventilation)
* according to indications. Shown is the implantation of a temporary, and subsequently - permanent pacemaker
* Taking into account the results of coronary angiography, determine the indications for myocardial revascularization (PCI (precutaneous coronary intervention)and / or coronary artery bypass grafting).
*Drug therapy should include the appointment of antiplatelet agents, statins, ACE inhibitors, after implantation of a pacemaker - β-blockers
Aspirin 300mg clopidogrel 300 mg, bisoprolol 10mg , statins (Atorvastatin 10-20mg)

How well did you know this?
1
Not at all
2
3
4
5
Perfectly
7
Q

Task 68
A 49-year-old patient complains of severe weakness, constant drowsiness, losing 6 kg in six months, gingival and nasal bleeding, abdominal distention, and itching. The patient’s condition is of moderate severity. In history: long-term alcohol abuse.
Examination was performed with the following results: jaundice of the skin, mucous membranes and sclera, vascular asterisks on the neck, chest, erythema Palmare, and Dupuytren’s contracture. There is atrophy of the muscles of the upper shoulder girdle, weight deficit (weight 58 kg with a height of 177 cm, BMI 17).
There are subcutaneous hematomas on the arms and legs.
There is abdomen distention. With percussion, there is fluid detected in the abdominal cavity.
The liver is palpated 4 cm below the costal arch, the edge is sharp and dense. The percussion dimensions are 13×11×6 cm. The percussion dimensions of the spleen are enlarged, 17×12 cm. Total protein 59 g/L, albumins 48%, globulins 52%, and gamma-globulins 28.5%.
Task
1. Suppose the most likely diagnosis.
2. Name the syndromes of internal organ damage.
3. Explain why you identified these syndromes.
4. Make a plan for additional examination of the patient and justify it.
5. What groups of medications should be administered to the patient?

A

1.diagnosis: Preliminary diagnosis: “liver cirrhosis (of toxic (alcoholic) origin), active phase, class “C”
by Child-Pugh (14 points) class c ,Decompensated portal hypertension ,varicose veins stage 1(spider veins ), Ascites grade 2, hepatoencephalopathy grade 2,hepatocellular insufficiency ,cholestatic syndrome,immune inflammatory syndrome,hemorrhagic syndrome
2.Name the syndromes of the internal organ
* Portal hypertension syndrome
* hepatocellular failure syndrome
* cholestatic syndrome
*hemorrhagic syndrome
*hepatoencephalopathy syndrome.
*Asthenovegetative syndrome
*cytolytic syndrome
3. Explain why you identified these syndromes
* The syndrome of portal hypertension is evidenced by an increase in the volume of the abdomen, the presence of effusion in the abdominal cavity with abdominal percussion, and an increase in the size of the spleen.,varicose veins ,
*The syndrome of hepatocellular failure :weight loss,skin changes ,muscle wasting and jaundice
* The cholestatic syndromeis manifested by the presence of itching, yellowness of the skin, sclera, mucous membranes.
* The presence of immune -inflammatory syndrome is evidenced by dysproteinemia, hypergammaglobulinemia, and enlarged spleen.
*cytolytic syndrome:weight loss,liver palms ,spider naevus ,fetor hepaticus
*Astenovegetative syndrome :fatigue drowsiness during the day and insomnia at night
*hemorrhagic syndrome: gingival bleeding,nasal bleeding and skin hemorrhages ,hematomas
4. Make a plan for additional examination and justify it
*Elastography method : fibroscan
*MRI is
* To confirm the diagnosis, it is necessary to perform fibroesophagogastroscopy with an assessment of the state of the veins of the esophagus and the cardiac part of the stomach, sigmoidoscopy with an assessment of the veins of the rectum,
* ultrasound examination (Doppler)of the abdominal organs to determine the size of the liver, its structure, the size of the spleen, the amount of ascitic fluid in the abdominal cavity and pleural cavities.
* . It is necessary to examine all functional liver tests ( ALAT, ASAT, bilirubin, prothrombin, cholesterol, alkaline phosphatase )
* a hemogram with the determination of the number of platelets
* perform a serological blood test for the presence of antibodies to hepatitis B, D and C viruses to exclude possible viral hepatotropic infections
* It is desirable to determine the level of alpha-fetoprotein to exclude hepatocellular carcinoma and the level of ferritin to exclude hemochromatosis of the liver.
Biopsy of liver
5. What groups of medications should be administered to the patient
* Non-selective beta blockers.
* Diuretics(furosemide 40mg)
* Albumin
* Hepatoprotectors
* Enzymes
* Prophylactic antibiotics
Spirinolactone 100 mg is the standard treatment
Ursodeoxychloric acid (15-20 mg per kg )for the itching skin
**
Additional information

  1. The most likely diagnosis for this patient is cirrhosis of the liver.
  2. The patient presents with several syndromes of internal organ damage, including jaundice, vascular asterisks, erythema Palmare, Dupuytren’s contracture, atrophy of the muscles of the upper shoulder girdle, weight loss, subcutaneous hematomas, and abdominal distention with fluid detected in the abdominal cavity.
  3. These syndromes are indicative of liver damage, specifically cirrhosis, which is a chronic liver disease characterized by fibrosis and the formation of nodules that replace healthy liver tissue. The vascular asterisks, erythema Palmare, and Dupuytren’s contracture are all associated with cirrhosis and result from changes in the blood vessels and connective tissue in the body.
  4. Additional examination of the patient should include liver function tests, imaging studies such as ultrasound or CT scan of the abdomen, and a liver biopsy to confirm the diagnosis of cirrhosis and assess the extent of liver damage. The patient should also be screened for hepatitis B and C, as these infections can lead to cirrhosis.
  5. Medications that may be administered to the patient include diuretics to reduce fluid buildup in the abdomen, vitamin K to address bleeding, and lactulose to manage hepatic encephalopathy. The patient may also require treatment for alcohol addiction, as continued alcohol abuse can worsen liver function and lead to further complications. In advanced cases of cirrhosis, liver transplantation may be necessary.
How well did you know this?
1
Not at all
2
3
4
5
Perfectly
8
Q

Task 48
A 72-yes-old pensioner has come to the district doctor with complaints of headaches, dizziness, tinnitus aurium, increase of arterial pressure to 210/120 mmHg.
In history: two years ago arterial pressure increased, the patient took Metoprolol, Hypothiazid,

however, this did not result in significant decrease of arterial pressure. Within six months, the patient notes fatigue when walking, pain in the legs which makes him stop (when walking less than 200 m). The patient is hospitalized to clarify the cause of the condition and select medication. Objective clinical examination: the condition is satisfactory. There is no edema. There is percussion pulmonary sound over the entire surface of the lungs. During auscultation, respiration is vesicular, without rale. The heart region is visually normal.
The heart borders are the following: the right one is 1 cm outwards from the right edge of the sternum, the upper one is the 2nd rib, the left one is 1.5 cm outwards from the left midclavicular line in the 5th intercostal space. Apex beat is in the 5th intercostal space, outward from the midclavicular line, diffuse. Cardiac sounds are muffled and rhythmic. The second sound above the aorta, systolic murmur on the aorta without conducting on the neck vessels. Pulse is 76 bpm, rhythmic. The vascular wall beyond the pulse wave is dense. Arterial pressure is 195/115 mmHg. The abdomen is soft and painless. The liver is along the edge of the costal arch.
A systolic murmur is heard above the abdominal aorta.
CVA tenderness test is negative. The pulsation is reduced in the arteries of the back of both feet. Clinical blood test was performed with the following results: hemoglobin 145 g/L, RBC count 4.5×1012/L, WBC count 7.9×109/L, ESR 12 mm/h, CI 1.0.
General urinanalysis was performed with the following results: specific gravity 1020, protein 0.033 g/L, WBC - units/hpf, single hyaline casts.
Biochemical blood test was performed with the following results: blood sugar 5.2 mmol/L (3.5- 6.1), cholesterol 7.2 mmol/L (4.0), urea 9.0 mmol/L (2.4-8.3), creatinine 0.13 mcmol/L (0.014- 0.44), potassium 5.4 mmol/L (3.4-5.3), sodium 135 mmol/L (130-156).
Isotope radiography was performed with the following results: moderate decrease in secretory and excretory functions of the right kidney.
The examination of the eye fundus was performed with the following results: retinal vascular angiopathy.
Echocardiography was performed with the following results: right ventricle 2.1 cm (normal value 2.3), interventricular septum 1.25 cm (normal value 1.0 cm), left ventricle posterior wall 1.25 cm (normal value 1.0 cm), left ventricular end diastolic diameter 5.8 cm (normal value 5.5), left ventricular end systolic diameter 3.7 cm (normal value 3.5).
The ejection fraction is 54% (normal value 60-80%). There is calcification of the mitral and aortic valves.
An ECG is presented.

Task
1. Identify the main syndromes (explain pathophysiological mechanisms of their appearance).
2. Make a differential diagnostic series.
3. Make a diagnosis and justify it.
4. Make a plan of examinations to specify the diagnosis.
5. Make a treatment plan.

A

1.Identify the main syndromes and explain Pathophysiology
*syndrome of arterial hypertension: Elevated blood pressure must be due to elevated cardiac output, elevated peripheral vascular resistance, or a combination of both. Each of these mechanisms is regulated, in turn, by hemodynamic, neural, humoral, and renal processes, all of which vary in their contribution from one individual to another . the predominant cause of hypertension tends to be elevated peripheral vascular resistance, often in combination with increased stiffness of the vessels, which manifests itself clinically as isolated systolic hypertension
* syndrome of cardiomegaly, mainly of the left sections: The hallmark pathophysiologic feature of dilated cardiomyopathy is systolic dysfunction. Several pathogenetic mechanisms appear to be operative. These include increased hemodynamic overload, ventricular remodeling, excessive neurohumoral stimulation, abnormal myocyte calcium cycling, excessive or inadequate proliferation of the extracellular matrix, accelerated apoptosis, and genetic mutations.
* intermittent claudication syndrome .: Peripheral vascular disease is primarily driven by progressive atherosclerotic disease resulting in the reduction of major organ blood flow and end-organ ischemia. The process of atherosclerosis is complex, with the involvement of numerous cells, proteins, and pathways
2: Make a differential diagnosis
* hyper aldosteronism
* coarctation of aorta
* renal artery stenosis
*chronic kidney disease
### Common are signs of arterial hypertension (increased blood pressure, tinnitus,
dizziness), increased blood pressure.
However, for isolated systolic hypertension, the diastolic pressure figures should be within the normal range. An increase in systolic blood pressure in old age is associated with an increase in OPSS ( the vascular wall becomes “rigid”, the elasticity of the vascular wall is reduced ).
Hypertension is characterized by an earlier onset (45-50 years), a long history of arterial hypertension. For vasorenal hypertension - old age, persistent increase in blood pressure, vascular noise, increased blood plasma cholesterol levels.
3. Make a diagnosis
Artherosclerosis of the aorta, renal vessels. Possible renal artery stenosis on the right. Vasorenal hypertension, stage 3 arterial hypertension (SAP >160, DAP > 100). Left ventricular hypertrophy. Stage 3A chronic kidney disease. Hypercholesterolemia. Intermittent claudication, stage IIA ischemia. Risk of cardiovascular complications 4 (very
high).
Justification: The main diagnostic signs: the patient’s age (72 years), monotonously high hypertension, the presence of vascular murmurs ( signs of atherosclerosis of the aorta : accent of the second tone over the aorta, systolic murmur of the abdominal aorta, increased plasma cholesterol level ), pain, fatigue in the legs when walking up to 200 meters, forcing the patient to stop, a decrease in pulsation in the arteries of the rear of the feet.
4.Make a plan of examinations to specify the diagnosis
* If possible, determination of plasma renin (from the renal veins)
* Rehberg’s test (to determine the function of the kidneys);
* Ultrasound of the kidneys;
* Excretory urography (to reveal the delay of the nephrographic effect
* Lipidogram; Contrast aortography with renal artery angiography (to reveal the presence of renal artery stenosis).
5. Make a treatment plan
* diet with restriction of animal fats, salt
* statins ;Atorvastatin 10mg (stabilization of atherosclerotic plaques, hypocholesteric effect);
* Norvasc 10 mg once a day in the morning (vasodilation, nephroprotective properties);
* small doses of diuretics : Indapamide 1.25 mg (a thiazide-like diuretic with a vasodilating effect);
* aspirin 300mg clopidogrel 300mg prevention of thrombosis in blood vessels.
* Active detection of signs of coronary atherosclerosis (coronary angiography) and BCA atherosclerosis (CDS).
* Consultation with a vascular surgeon for diagnosis and determination of indications for surgical treatment (nephrectomy, or reconstructive surgery on the renal vessels - removal of atherosclerotic plaques from the renal arteries, or angioplasty).

How well did you know this?
1
Not at all
2
3
4
5
Perfectly
9
Q

Task 53
A 42-year-old patient, a housewife, has come to the district doctor with complaints of feeling general weakness for several years and a body temperature increase to subfebrile without any cause.
A month ago, after an acute respiratory disease, there appeared moderate pain and swelling of the 2nd and 3rd metacarpophalangeal joints, the 2nd, 3rd and 4th proximal interphalangeal joints of both hands; joint stiffness lasting for 3 hours in the morning.
Joint syndrome is accompanied by an increase of body temperature to 37.3°С.
The patient has difficulty doing the housework.
Objective clinical examination: the skin is of normal color and clean. Respiration is vesicular, without rale. The left border of relative cardiac dullness is along the midclavicular line. Cardiac sounds are rhythmic. Arterial pressure is 130/80 mmHg.
There is a defiguration of the abovementioned joints due to exudative and proliferative phenomena, diffuse painfulness, active and passive movements are limited and painful.
Clinical blood test was performed with the following results: WBC count 9.0×109/L, ESR 35 mm/ h.
Biochemical blood test showed that rheumatoid factor (RF) was 1:80.
Anti-cyclic Citrullinated Peptide Antibody (anti-CCP) 375.8 U/ml.
Hand radiograph was performed with the following results: periarticular osteoporosis, joint space narrowing, and single bone usuras.
Task
1. Suppose the most likely diagnosis.
2. Justify the diagnosis.
3. Make and justify a plan for additional examination of the patient.
4. What will be the treatment strategy for this disease?
5. What basic medication is advisable to prescribe in this case and why?

A

1.Diagnosis: Advanced erosive seropositive rheumatoid arthritis(because there is rheumatoid factor positive in blood), ACPAs+, moderate activity (DAS28=5.0), radiological stage II, functional class of joint insufficiency II. Advanced se- ropositive rheumatoid arthritis, ACPAs+, moderate activity (DAS28=5.0), erosive (radiolog- ical stage II), FC III.
2. Justify the diagnosis
* The diagnosis of “rheumatoid arthritis” was made on the basis of typical signs of articular syndrome: morning stiffness, swelling of the metacarpophalangeal and proximal interphalangeal joints. X-ray data of the hands: periarticular osteoporosis, narrowing of the joint space, usury. Detection of high numbers of rheumatoid factor, antibodies to cyclic citrullinated peptide and ESR. The activity of the process is determined using the DAS28 index. Functional deficiency class - due to the restriction of homework. It is advanced because she has been suffering the disease for several years and it’s seropositive because rheumatoid factor is positive in blood. Das28 is just a criteria for checking the 28 joints affected by rheumatoid arthritis which indicates the disease activity thus score 5.0 is moderate activity
3.make and justify additional examination plan for patient
* . The patient is recommended to have an X-ray examination of the lungs
* ultrasound examination of the abdominal cavity and kidneys
* EchoCG and ECG
* Determination of the general analysis of urine.
* Blood creatinine, ALT, AST, bilirubin
* Examination by an ophthalmologist, neurologist.
4. What will be the treatment strategy
* After examination, the patient needs to prescribe basic therapy - Methotrexate 15 mg per week, Folic acid 5 mg per week.
* To relieve pain, it is necessary to prescribe NSAIDs (for example, celecoxibs retard 400mg , or diclofenac 100mg 2 times a day), in combination with Omeprazole 20 mg 1 time a day.
*Because methotrexate takes time to kick in we can prescribe methotrexate + glucocorticoids for 2-3 months then Wait for the effectiveness of methotrexate then gradually reduce glucocorticoids and then stop it use alternating regimen of glucocorticoids
5.what basic medication is advised
* The first-line drug - Methotrexate, since this drug most effectively controls the autoimmune inflammatory process in RA, slows down the X-ray destruction of bone tissue, prevents the development of systemic manifestations of the disease, is well tolerated. But additional drug such as folic acid will help to prevent side effects of methotrexate

How well did you know this?
1
Not at all
2
3
4
5
Perfectly
10
Q

Task 54
Patient Yu., 53 years old, has come to the district doctor with complaints of weakness, pain throughout the spine (mainly at night), in both hip and knee joints, morning stiffness up to 40 minutes, which passed after exercise.
History: the patient has been ill since the age of 35, pain in the loin and sacrum (especially at night) appeared, then there appeared stiffness after a short morning exercise, and the body temperature increased to 37-37.5°С.
Objective clinical examination: the condition is satisfactory; the position is active. Arterial pressure is 150/100 mmHg. Heart rate is 74 bpm. Respiration rate is 18 brpm. Body temperature is 37.3°С.
There is a severe thoracic kyphosis and cervical hyperlordosis, smooth lumbar lordosis, pain during palpation along the spine, paravertebral lines and hip joints. Active and passive movements cause pain.
Tests were performed with the following results: Kushelevsky’s symptom I and II positive, Thomayer test - 65 cm, Forestier 22 cm, chin-chest test 5 cm, chest excursion test 100-96 cm (4 cm).
Radiograph of the spine and ileosacral joints was performed with the following results: deforming spondylosis, bilateral sacroiliitis, grade III.
Clinical blood test was performed with the following results: RBC count 4.2×1012/L, Hb 122 g/L, platelet count 220×109/L, WBC count 10×109/L, band neutrophils 1%, segmented neutrophils 63%, monocytes 4%, eosinophils 2%, lymphocytes 30%, ESR 18 mm/h.
Immunological tests were performed with the following results: C-reactive protein 5mg/L, negative rheumatoid factor.
General urinanalysis is within the range of normal limits.
Task
1. Suppose the most likely diagnosis.
2. Justify the diagnosis.
3. Make and justify a plan for additional examination of the patient.
4. What will be your treatment strategy for this disease?
5. What medications are more preferable for this patient taking into consideration
the extra-axial manifestations of the disease?

A

1.Diagnosis: Ankylosing spondylitis, late stage, moderate activity (BASDAI 3,2). Bilateral sacroiliitis, grade III. Bilateral coxarthrosis. Extra-axial manifestations (knee joint oligoarthritis). Functional Class III. Complications: cervical-thoracic kyphosis (occiput to wall distance - 22 cm)
2.Justify the diagnosis
* The diagnosis of “ankylosing spondylitis (AS)” was made on the basis of the inflammatory nature of the pain in the spine, changes in the corresponding clinical tests (Kushelevsky, Thomayer, Forestier) and the detection of ileosacral joints of stage III sacroiliitis on the radiograph. The extended stage was set on the basis of the duration of the course of the disease and the absence of ankylosis in various parts of the spine. The value of the BASDAI (Bath ankylosing spondylitis disease activity index is calculated based on 1: presence of HLA- B27,positive buttock pain which resolves with exercise and MRI and x ray evidence of sacroiliac involvement and BASFI indices based on the calculation of the corresponding medical calculators and used as a criteria to assess the progress of therapy of the disease . Functional joint failure was established on the basis of changes in the physiological curves of the spine and limitation of the spine’s mobility
3. Make and justify a plan for additional examination
* The patient is recommended to x-ray and ultrasound of the hip joints to determine the presence of coxitis in the patient.
* Examination by an ophthalmologist to identify the manifestation of eye damage.
* Performing an ECG and EchoCG to detect aortitis or other manifestations of the involvement of the heart in the pathological process.
* Determination of HLA-B27, antibodies from the major histocompatibility complex of human leukocytes to the B27 antigen, which indicate the genetic nature of the disease.
4. Treatment strategies
* All patients need the appointment of NSAIDs and exercise therapy.
* Taking NSAIDs should be constant and long-term. NSAIDs are basic drugs for the treatment of AS (for example, Diclofenac sodium 100 mg 2 times a day or Etoricoxib 90 mg 1 time a day).
* In the absence of the effect of taking NSAIDs - the decision on the appointment of tumor necrosis factor alpha inhibitors (Infliximab, Adalimumab, Golimumab).
* When the presence in the patient’s phenomena peripheral arthritis should be the appointment of sulfasalazine. With its ineffectiveness, the decision on the appointment of genetically engineered biological drugs from the group of tumor necrosis inhibitors alpha (TNF alpha), since

How well did you know this?
1
Not at all
2
3
4
5
Perfectly
11
Q

Task 55
A 64-year-old patient has come to the district doctor with complaints of swelling and pain in the right ankle joint and small joints of the right foot, redness of the skin above them, and restriction of movement in these joints.
History: the patient has been suffering from sudden pain attacks in right foot joints for about 8 years, when the patient was feeling relatively well and intense pain appeared in the first toe of the right foot at night. Further on, the arthritis repeatedly recurred. The pain in the right ankle joint appeared during the last six months. Repeatedly, yellowish brown concrements of up to 3-4 mm in size were removed with the urine.
Objective clinical examination: normal constitution, high-calorie diet. Near the cartilaginous part of the auricles, painless dense formations of 0.3x0.2 cm are palpated, the formations are whitish at the bend. There is bone deformation in the area of the 1st and 2nd metatarsophalangeal joints of the right foot, accompanied by swelling, skin reddening and increase of body temperature over the same joints. The right ankle joint is swollen and painful during palpation. The skin above the joint is shiny, of bluish purple color and hot. Arterial pressure is 170/105 mmHg. Heart rate is 84 bpm. The borders of relative cardiac dullness are extended to the left by 2 cm from the midclavicular line. Cardiac sounds are muffled and rhythmic.
Uric acid 780 mmol/L, blood cholesterol 6.7 mmol/L, triglycerides 2.7 mmol/L, HDL 1.0 mmol/L, glucose fasting 6.2 mmol/L, 2 hours after taking 75 g of glucose 6.4 mmol/L.
Task
1. Suppose the most likely diagnosis.
2. Justify the diagnosis.
3. Make and justify a plan for additional examination of the patient.
4. What will be your treatment strategy during the acute attack and between the attacks?
5. What medications are more preferable for the correction of the arterial hypertension and hypercholesterolemia? Why?

A

1.Diagnosis: Primary disease. Chronic tophaceous gout: acute oligoarthritis, nephrolithiasis; subcutane- ous tophi, hyperuricemia.
Concomitant disease. Essential hypertension (stage II: systolic 160 or greater, diastolic 100 or greater), arterial hypertension (grade II: 140-159/90-99). Left ventricular hypertrophy. Hyperlipidemia, hyperuricemia, obesity; risk 3. Target BP< 130/< 80 metabolic syndrome
2.justify the diagnosis
* The diagnosis of “gout” is made on the basis of typical signs of articular syndrome: paroxysmal inflammation of the joints of the foot, intense pain, swelling of the joints, in addition, the recurrent nature of the course and the presence of tofuses are important.The condition is chronic because patient has been suffering the disease for 8 years ,also presence of subcutaneous tophias we can see in the auricles, and finally hyperuricemia due to high level in blood (780mmol/l) normal range is 0.24-0.51mmol/l)
The diagnosis of hypertensive disease (HD) was established on the basis of the patient’s complaints of instability of blood pressure, the establishment of the degree of hypertension is based on the BP figures measured during admission.
The stage of hypertension is established based on the presence of damage to target organs (heart). The degree of risk of cardiovascular complications is based on the presence of metabolic syndrome.
The diagnosis of “metabolic syndrome” was established on the basis of obesity, hypertension, dyslipidemia patient has high cholesterol level in blood (6.7mmol/l. Normal cholesterol levels is 5.3mmol/l(an increase in the level of triglycerides and a decrease in the level of HDL), and fasting hyperglycemia.
Patient also had left ventricular hypertrophy due to the borders of the relative cardiac dulled are extended to the left by 2cm from the midclavicular line normal is 1cm
3.Make and justify a plan for additional examination of the patient
* The patient is recommended to x-ray examination of the feet
* polarization microscopy of synovial fluid to visualize uric acid crystals,
* ultrasound of the kidneys to determine kidney damage,
* daily monitoring of blood pressure to assess the stability of the increase in blood pressure, daily blood pressure profile
* conducting an ECG; echocardiography to assess myocardial wall thickness, diastolic and systolic function; laboratory test
* creatinine to determine GFR and stage CKD.
4. What will be your treatment strategy during acute attack and between the attacks
* All patients need to follow dietary recommendations (limiting foods containing purines (meat), limiting fat in foods
* prohibiting alcohol intake
* To relieve acute articular syndrome, 3 groups of drugs are used: NSAIDs (for example, Diclofenac 100 mg 2 times a day after meals), glucocorticosteroids (Diprospan 1.0 locally) or Colchicine 1 mg 3 times a day.
* In the interictal period: it is necessary to continue adhering to the diet, add Allopurinol 300 mg per day to therapy until uric acid normalizes, followed by a decrease in the dose to the maintenance dose - 100 mg per day, NSAIDs - for pain.
**
first line for acute attack is colchicine 0.5 mg 3 times a day + diclofenac 100 mg
* in case of multiple joint lesions Glucocorticoids prednisone 40-60 mg systemic use on the first day and gradually reducing the dose 5 mg till you stop because of side effects
5. What medications are more preferable
*uricosuric drugs such as probenecid and benzbromarone
*Febuxostat(xanthine inhibitors)
*uricases (pegloticase)
* Antagonists of receptors to angiotensin II for the correction of blood pressure and Atorvastatin 10mg to reduce hyperlipidemia, since these drugs have a uricosuric effect.
* In addition, the choice of a drug from the group of angiotensin II receptor antagonists is based on its nephroprotective properties.
* From the same position, it is justified to use a drug from the group of ACE inhibitors as an antihypertensive agent. The most proven nephroprotective properties from the group of ACE inhibitors are possessed by Ramipril, from the group of angiotensin II receptor antagonists - Losartan.

How well did you know this?
1
Not at all
2
3
4
5
Perfectly
12
Q

Task 47
A 39-year-old man complains of dyspnea, palpitations and cough at a local doctor’s appointment.

The patient notes having slight dyspnea and periodic palpitation for 5 years. About a week ago, the patient got angina with high fever and cough. The night before going to the doctor, the patient could not sleep because of severe dyspnea, which aggravated in the horizontal position.
In adolescence, the patient often had angina which was accompanied by pain in large joints. The patient was observed by a neurologist for chorea.
Objective clinical examination: orthopnea, acrocyanosis; respiration rate is 28 brpm, edema of the lower extremities, with lifting apex beat. Pulse is weak, arrhythmic, 96 bpm.
According to auscultation, heart rate is 110 bpm. Arterial pressure is 100/60 mmHg. Body temperature is 37.4 °С. The liver is enlarged and slightly painful during palpation.
With percussion, the heart is enlarged to the left and right. There is fine bubbling rale in the lower parts of the lungs.
At heart auscultation, there is arrhythmia with no periods of regular rhythm. In the apex, there is a three-part melody with a low dull additional component, the second sound is on the pulmonary artery. A three-part melody is auscultated in Botkin-Erb’s point. There is systolic and protodiastolic murmur at the apex. At the apex systolic murmur increases at exhalation and is conducted to the axillary region.
Clinical blood test was performed with the following results: ESR 32 mm/h, WBC count 11300 in 1 mm3.
Biochemical blood test showed C-reactive protein (++++). Diphenylamine test is 0.500 (N up to 0.200).
Task
1. Your tentative diagnosis.
2. Criteria for the main diagnosis.
3. What complications of the underlying disease do you suppose?
4. Make a plan for additional examination of the patient and justify it. 5. What is the treatment strategy and choice of medication?

A

1.Diagnosis: Recurrent rheumatic fever: carditis. Combined heart disease: stenosis and mitral insufficien- cy. Atrial tachysystole. Chronic heart failure stage IIB, Functional Class IV. Cardiac asthma.Hepatomegaly
2.criteria for the main diagnosis
* The patient has a rheumatic history, clinical and laboratory data, indicating the activity of the rheumatic process against the background of the transferred sore throat. The data of an objective examination and auscultation of the heart indicate the formation of a combined mitral heart disease: stenosis and insufficiency of the mitral valve. The nature of the pulse, the presence of a pulse deficit indicate atrial , mitral valves is usually associated with rheumatic fever,the murmurs is a typical description of the case
*Patient also has chorea when he visited the neurologist
* chronic heart insufficiency due to presence of symptoms such as edema,bubbling rales in the lower lungs
* in the laboratory results we can see see increased ESR 32mmhr, C-reactive proteins (++++)
*Patient also had arthragias which is indicative of this disease
*cardiac asthma; Cardiac asthma is not a form of asthma. It’s a type of coughing or wheezing that occurs with left heart failure. Depending on how severe the symptoms are, this wheezing can be a medical emergency. Heart failure can cause fluid to build up in the lungs (pulmonary edema) and in and around the airways.
3. What complications of the underlying disease do you suppose
* Bacteria endocarditis
* stroke can develop from atrial fibrillation
*ruptured heart valves and formation of vegetations on the valves
4. Make a plan for additional examination of the patient and justify it
* The patient is recommended: to identify the presence of streptococcal infection - a smear from the throat for bacteriological examination, determination of the ASL-O titer in the blood serum;
* to determine the nature of the organic lesion of the mitral valve, to assess the degree of mitral stenosis and mitral insufficiency.
* To assess the nature of the remodeling of the left ventricle, left atrium, right ventricle and right atrium by transthoracic echocardiography.
* Exclude the presence of thrombi in the atria by transesophageal echocardiography.
* Perform a chest x-ray.
5.what is the treatment strategy and choice of medication
* Antibiotic therapy. long-acting penicillin G benzathine An injection of 0.6-1.2 million units administered intramuscularly (IM) every 28 days rather than other regimens.
* Non-steroidal anti-inflammatory drugs
* To control the heart Digoxin is prescribed
*Aspirin 300 mg and clopidogrel 300mg for prevention of stroke
* With a decrease in the ejection fraction - Bisoprolol or Carvedilol, Spironolactone, ACE inhibitors (Enalapril or Lisinopril), a loop diuretic (Torasemide or Furosemide).
* For the relief of cardiac asthma - intravenous administration of nitrates. Consultation with a cardiac surgeon.
* Surgical treatment - mitral valve replacement - is indicated after the activity of the rheumatic process subsides and the degree of circulatory failure decreases.

How well did you know this?
1
Not at all
2
3
4
5
Perfectly
13
Q

Task 56
A 65-year-old retired woman has come to the district doctor with complaints of pain in knee and hip joints, distal interphalangeal hand joints, and in the lumbar spine, which appears during physical activity and moving. The pain appears more often in the evening and passes at rest. The patient also feel morning stiffness lasting up to 20 minutes. The patient cannot determine for how long she has been ill. The patient notes a gradual increase, during 5-7 years, of the abovementioned pain in the listed joints and lumbar spine. Previously the patient was neither examined nor treated. For the last 20 years, the patient has been working as a cleaner at school as she continues working in retirement. The patient notes frequent intense physical activity. Objective clinical examination: the patient is 160 cm tall and weighs 80 kg. The skin is of flesh colored with high humidity. There is no edema. The knee joints are deformed. The skin above these joints is of normal color. The body temperature is regular there. There is a slight restriction of joint movement due to painfulness. There is crepitus in the knee joints during active movements. There is pain during palpation and restriction of movement of the distal interphalangeal joints. The joints are shown in the picture below. The other joints are not changed. During palpation, the patient feels pain in the spinous and transverse processes of the lumbar spine. The movement is restricted there due to painfulness.
Respiration is vesicular, without rale. Pulse is rhythmic, satisfactory and tense, 78 bpm.
Arterial pressure is 135/80 mmHg. The borders of relative cardiac dullness are within the normal range.
Cardiac sounds are muffled and rhythmic, without noise. The abdomen is soft and painless. Clinical blood test was performed with the following results: hemoglobin 130 g/L, WBC count 6.7×109/L, eosinophils 3%, band neutrophils 4%, segmented neutrophils 57%, lymphocytes 30%, monocytes 6%, ESR 20 mm/h.
Knee radiograph was performed with the following results: joint space narrowing, subchondral
sclerosis, and marginal osteophytes.
Hand radiograph was performed with the following results: multiple osteophytes of distal interphalangeal joints, moderate joint space narrowing, osteosclerosis.
Task
1. Suppose the most likely diagnosis.
2. Justify the diagnosis.
3. Make and justify a plan for additional examination of the patient.
4. Make a treatment plan for this patient. Justify your answer.
5. Determine the prognosis of the disease and the patient’s ability to work.

A

1.Diagnosis: Primary disease: primary generalized osteoarthritis: bilateral gout, grade 3 according to the Kellgren-Lawrence grading system, bilateral coxarthrosis, arthrosis of distal interphalangeal joints of the hands (DIP joints), secondary synovitis of DIP joints and knee joints. Underly- ing condition: class 1 obesity (BMI - 31)
2.justify the diagnosis:
* Osteoarthritis: mechanical pain, stiffness within 20 minutes, deformation of the joints for
account of bone growths. X-ray stage: narrowing of the joint space, osteophytes, osteosclerosis.
Functional class: difficulty in professional activity. Obesity I degree (BMI - 31.3 kg / m 2 ).
3. Make and justify a plan for additional examination for the patient
* The patient is recommended: to clarify the degree of activity - determination of acute phase parameters (fibrinogen, CRP, proteinogram).
* X-ray of the lumbar spine
* Ultrasound of the knee joints (additional examination of the musculoskeletal system).
4. Make a treatment plan for this patient and justify the answer
* Basic therapy (chondroprotectors) - chondroitin sulfate 500mg 2 time a a day glucosamine sulfate 1500mg two times a day, Dona, Structum, Artra, Alflutop).
* NSAIDs (Movalis, Naproxen, Nimesulide).
* HA intra-articular in the presence of synovitis.
Intra-articular hyaluronic acid preparations.
* Antiplatelet agents and antagonists of Ca to improve microcirculation in the subchondral parts of the bones and in the synovium
* Exercise therapy, massage, physiotherapy (magnetotherapy, UHF, ultrasound).
Local therapy (NSAIDs, Dimexide) Treatment of concomitant diseases (correction of body weight).
Spa treatment.
##It is forbidden to walk for a long time, standing for a long time, carrying weights; frequent ascents and descents of stairs are not recommended.
5. Determine the prognosis of the disease and the patient ability to work
* Life prognosis is favorable. Possible disabling damage to the knee and hip joints. At the time of exacerbation - temporary disability.

How well did you know this?
1
Not at all
2
3
4
5
Perfectly
14
Q

Task 57
A 35-year-old woman, an accountant, has come to the district doctor with complaints of general malaise, weakness, weight loss of 5 kg in 3 months, migratory pain in the interphalangeal hand joints and ankle joints, irregular red spots on the face, and subfebrile body temperature. The patient considers herself ill for about 6 months. The disease onset is gradual. There were pain in the interphalangeal joints, swelling and hyperemia, alternately on the right and left hands.
Weakness and malaise appeared gradually. The patient did not seek medical help. Two months ago she was on vacation in Thailand. Upon return, the patient noted a deterioration in her health, which involved an increase of weakness, accompanied by pain in ankle joints. The patient lost weight for no apparent reason. The body temperature increased to 37.5°С without shivering. The patient noted red spots on her face and for a long period of time considered them to be the consequences of sunburn. During the next scheduled medical examination, anemia and changes in the urine were detected, and therefore the patient came to the district doctor.
Objective clinical examination: skin and visible mucous membranes are pale, red rash on the bridge of the nose and cheeks of irregular shape, hair is dull, brittle with areas of alopecia. Subcutaneous fat is almost absent. There is no edema. Submandibular, axillary and inguinal lymph nodes are palpated 0.5-1cm in diameter. Their consistency is soft. The lymph nodes are not tender to palpation. Muscle tone and strength are normal, the same on both sides. The parts of the skeleton are proportional, the bones are painless when palpated and tapped. There is swelling proximal and distal interphalangeal hand joints and ankle joints, painful during moving, with local hyperemia. The thorax is normosthenic, symmetrical and evenly moves during breathing. With percussion, there is a clear pulmonary sound over the lungs. Respiration is vesicular, without rale. Pulse is rhythmic, 78 bpm. Arterial pressure is 120/80 mmHg. Apex beat is not detected visually and during palpation. The borders of relative cardiac dullness are the following: the right one is 0.5 cm from the right edge of the sternum; the upper one is the lower edge of the 3rd rib; and the left one is 1 cm inwards to the midclavicular line.
During auscultation, cardiac sounds are muffled and rhythmic. Heart rate is 78 bpm. There is edema and hyperemia of the vermilion border. The abdomen is symmetrical, soft and painless in all the regions during surface and deep palpation. The liver is not enlarged. CVA tenderness test is negative on both sides, the kidneys are not palpated, palpation in their projection is painless. Clinical blood test was performed with the following results: hemoglobin 100 g/L, RBC count 2.9×1012/L, CI 0.9, WBC count 2.7×109/L, eosinophils 4%, band neutrophils 3%, segmented neutrophils 52%, lymphocytes 35%, monocytes 6%, ESR 30 mm/h, platelet count 98×10%. General urinanalysis was performed with the following results: light yellow, clear, acidic pH, specific gravity 1016, protein 0.3 g/L, glucose negative, WBC 1-2/hpf, renal epithelium 2-4/hpf, RBC 3-4/hpf, hyaline casts 5-8/lpf, waxy casts 2-3/lpf, urine sodium negative.
Biochemical blood test was performed with the following results: total bilirubin 38.8 mcmol/L, direct bilirubin 8.2 mcmol/L, indirect bilirubin 30.6 mcmol/L, creatinine 0.09% mmol/L, glucose 4.3 mmol/L, AST 10.0 mmol/L, ALT 19.0 mmol/L, cholesterol 4.0 mmol/L, potassium 3.9 mmol/ L, total protein 86 g/L, albumins 45%, α1 3.5%, α2 10.5%, β 13,6%, γ 27.4%, fibrinogen 6.2 g/L, C-reactive protein + + +, seromucoid 0.32 g/L.
There were antibodies to DNA, antinuclear factor, LE-cells 6 per 1000 WBC detected.
ECG was performed with the following results: sinus rhythm, heart rate 64 bpm. The electric axis is tilted to the left. There are diffuse dystrophic changes in the left ventricular myocardium.
Chest radiograph did not reveal any pathology.
Hand and ankle radiograph did not show any pathology.
Task
1. Suppose the most likely diagnosis.
2. Justify the diagnosis.
3. Make and justify a plan for additional examination of the patient.
4. Make a treatment plan for this patient. Justify your answer.
5. Determine the prognosis of the disease and the patient’s ability to work.

A

1.Diagnosis: Subacute systemic lupus erythematosus (Subacute cutaneous lupus erythematosus (SCLE) is a nonscarring, non–atrophy-producing, photosensitive dermatosis. SCLE commonly develops in sun-exposed areas, including the upper back, shoulders, extensor arms, neck, and upper torso, while the face is often spared), high activity (SLEDAI - 12 points): polyarthritis, photosensitization, skin and appendage damage (butterfly-shaped rash on the face, cheilitis, alopecia) nephritis, CKD?, hematological disorders (thrombocytopenia, leukope- nia, mild normochromic hemolytic anemia?), anti-DNA+
2. Justify the diagnosis
* The diagnosis was made on the basis of the identified syndromes: cutaneous, articular, febrile, lymphadenopathy, anemic, urinary.
3. Make and justify a plan for additional examination
* The patient is recommended:
-EchoCG (detection of heart damage);
* determination of GFR by calculation method (determination of chronic renal failure and chronic kidney
disease
* kidney biopsy (gold standard of diagnosis).
* Immunoblogging for systemic immune markers, Sm antigen, antiphospholipid antibodies
4. Make a treatment plan for this patient and justify your answer
* Hospitalization in a specialized department.
* A diet high in polyunsaturated fatty acids, calcium, vit. D.
* Hydroxychloroquine 200-400mg daily is the cornerstone of lupus therapy. Methylprednisolone pulses followed by low–medium doses of prednisone rapidly control most moderate– severe flares.
* Cytotoxic drugs (cyclophosphamide 1 g / m 2 bolus per month for at least 6 months, or methotrexate 15 mg / week s.c.), HA (high dose pulse therapy),
* NSAID, plasmapheresis, genetically engineered biologicals (Rituximab or Orensakh).
5. Determine the prognosis of the disease and patient’s ability
* The prognosis is relatively favorable: it is possible to achieve stable clinical and laboratory remission with lifelong maintenance therapy.
It is necessary to define the disability group (II)

How well did you know this?
1
Not at all
2
3
4
5
Perfectly
15
Q

Task 58
A 44-year-old woman, a programmer, was admitted to the Department of Internal Medicine with complaints of increasing general weakness, pain, restricted movement and numbness mainly of fingers on both hands, slight dry cough, palpitation during physical activity, decreased appetite,dry mouth, and difficulty chewing and swallowing.
The patient considers herself ill since the age of 32 when after severe hypothermia she began to notice numbness of fingers on both hands during washing. Then the duration of numbness gradually increased. It began to occur even with a slight decrease in the atmospheric temperature. At the same time, the patient developed increased sweating in both palms. Over six months, the patient gradually developed general weakness, decreased appetite, which was the reason the patient came to the district clinic. Clinical blood test was performed as well as chest radiograph, FGD, however, the cause of asthenization was not found. Numbness of the fingers persisted. Gradually the patient developed a restriction of finger mobility and pain in the nail phalanges of the hands.
At a second examination a year later, the patient was diagnosed with rheumatoid arthritis. The patient was prescribed Prednisolone 20 mg a day, which stopped the pain, but did not relieve numbness in the hands. The patient decided to stop taking Prednisolone on her own. At the age of 42, after exposure to the sun, the patient began to notice difficulty swallowing, dry cough and palpitations when walking. In the last six months, the pain in the nail phalanges has greatly aggravated when the patient tried to type on a computer keyboard.
Objective clinical examination: the skin is evenly hyperpigmented. The skin of the hands and fingers is cold, hard, evenly thick. The skin cannot be taken in a fold. There are no wrinkles above the interphalangeal joints. The nail phalanges of all fingers are shortened. The muscles of the shoulder and pelvic girdle are atrophic. The muscles of the forearms are dense and rigid. The range of motion (both passive and active) is sharply reduced in the interphalangeal joints. Respiration is shallow, respiration rate is 24 brpm. There is a decrease in the lung function and symmetrical weakening of the vesicular breathing in the lower parts. Arterial pressure is 110/70 mmHg. The cardiac sounds are arrhythmic, 88 bpm, with single extrasystoles; the first sound above the apex and base of the xiphoid process is diminished. The abdomen is soft and painless to surface and deep palpation.
Clinical blood test was performed with the following results: RBC count 3.1×1012/L, hemoglobin 95 g/L, CI 0.85, WBC count 15.3×109/L, eosinophils 2%, band neutrophils 8%, segmented neutrophils 72%, lymphocytes 17%, monocytes 1%, ESR 27 mm/h.
Biochemistry blood test was performed with the following results: total protein 75 g/L, albumins 40%, globulins 60%, 1 - 3.8%, 2 - 12%, 32.2%, fibrinogen 6.6 g/L, C-reactive protein (++). Repeated studies of LE cells in blood are negative.
Chest radiograph was performed with the following results: a honeycomb lung, pneumosclerosis, mainly in the lower zones.
Hand radiograph: osteolysis of the distal phalanges and epiphyseal osteoporosis.
Attachment:
Patient’s ECG:
Task
1. Suppose the most likely diagnosis.
2. Justify the diagnosis.
3. Make a plan for additional examination of the patient and justify it.
4. Determine the treatment plan for this patient. Justify your answer.
5. Determine the prognosis of the disease and the patient’s ability to work.

A

1.Diagnosis: Chronic generalized systemic sclerosis, active phase II, limited form: Raynaud syndrome, induration and hyperpigmentation of skin, telangiectasias, pathology of esophagus, articular muscle syndrome with contractures, acro-osteolysis, diffuse fibrosing alveolitis with the
formation of a cellular lung. Respiratory failure?; old myocardial infarction (chronic pulmo- nary heart?), complete right bundle branch block, extrasystole. Chronic heart failure, stage I. anemia
2.justify the diagnosis
* The diagnosis was made on the basis of the identified syndromes: Raynaud’s syndromes, scleroderma, articular, dysphagia, respiratory failure, anemic, immune disorders
3. Make an additional plan for examination
* The patient is recommended (to confirm and clarify the diagnosis): wide-field capillaroscopy of the nail bed,
* immunogram,
* R-graphy of the esophagus with contrast,
* skin biopsy,
* EchoCG, CM ECG, FVD, FGDS, fundus, antibodies to DNA, ANF, aticenter antibodies, anti Scl 70
*coagulogram
*Blood biochemistry
*Immunological studies
4. Determine treatment plan
* Treatment of vascular complications: calcium antagonists , nifedipine 30-120 mg a day (Amlodipine 5-20 mg a day), Pentoxifylline 400mg , Losartan 25-100 mg / day;
* for the healing of digital ulcers - Prostaglandin E(6-10 ng/kg), Sildenafil (Viagra)
* suppression of the progression of fibrosis: D-penicillamine 250-500 mg / day;
* if interstitial lung disease is confirmed, then combination therapy with Prednisolone + Cyclophosphamide; given the defeat of the esophagus, fractional meals are necessary, the last meal is no later than 18 hours,
* with severe dysphagia - Metoclopramide 10 mg 3-4 r / day, in the presence of reflux esophagitis - Omeprazole 20 mg / day.
Spa treatment is contraindicated
* Massage, exercise therapy in the absence of disease activity. The prognosis is poor, associated with visceral lesions.
5. Determine the prognosis of the disease and the patients ability to work
* The patient is incapacitated, the definition of a disability group (II) is required.

How well did you know this?
1
Not at all
2
3
4
5
Perfectly
16
Q

Task 59
A 45-year-old woman, a sales person, has come to the clinic with complaints of suffocation
attacks and dyspnea which occurred after physical activity and spontaneously at night, and chest discomfort. The patient got sick for the first time after severe pneumonia which occurred 11 years ago. Then the attacks recurred after physical activity and during common cold. The patient stopped suffocation attacks with inhalation of Salbutamolum (3-4 times a day).
In history: community-acquired bilateral bronchopneumonia and acute appendicitis. The patient denies having hereditary allergic diseases or any history of allergy in the family. There were no blood transfusions. The patient does not have bad habits.
Objective clinical examination: the patient’s condition is satisfactory; the consciousness is clear. The skin and mucous membranes are clean and of normal color. The tongue is moist. Lymph nodes are not enlarged. In the lungs, there is bandbox sound. During auscultation, respiration is harsh, with dry rale over all lung fields, whistling at forced exhalation. Respiration rate is 18 brpm.
The borders of the heart are not changed. Cardiac sounds are muffled and rhythmic. Arterial pressure is 140/90 mmHg.
Pulse is 69 bpm, full and tense.
The abdomen is soft and painless. The liver and spleen are not palpated. Physiological functions are normal.
Blood test was performed with the following results: hemoglobin 12.6 g/L, RBC count 3.9×1012/L, WBC count 9.5×109/L, band neutrophils 3%, segmented neutrophils 63%, eosinophils 5%, monocytes 6%, lymphocytes 13%, ESR 19 mm/h.
Biochemical blood test was performed with the following results: total bilirubin 5.3 mcmol/L, total protein 82 g/L, urea 4.7 mmol/L.
Urinanalysis was performed with the following results: specific gravity 1028, protein negative, epithelium 1-3/lpf.
Sputum analysis: crudum, odorless. Microscopy was performed with the following results: WBC 5-6/hpf, eosinophils 10-12/hpf, bronchial epithelial cells, single alveolar macrophages. TB is negative (3 times).
A chest radiography was performed with the following results: increased transparency of the lung fields, flattening of the diaphragm and its low position. The pulmonary pattern is enhanced. The roots of the lungs are enlarged, the shadow is enlarged as well. The transverse cardiac shadow is enlarged.
Task
1. State the expected provisional diagnosis.
2. Justify the diagnosis.
3. Make a plan for additional examination of the patient.
4. Differential diagnosis.
5. Treatment plan (identify the necessary groups of medications).

A

1.Diagnosis: Bronchial asthma of complex origin (infection-dependent, exercise-induced), uncontrolled, moderate severity, in the acute phase, respiratory failure 0.Arterial hypertension grade 1 stage 1 No cardiovascular risk , community acquired bilateral bronchopneumonia and acute appendicitis
2.Justify the diagnosis
* The diagnosis was made on the basis of the patient’s complaints of asthma attacks and shortness of breath after physical exertion and spontaneous at night, for chest discomfort; anamnesis of life (attacks are repeated after exercise and during colds; attacks of suffocation were removed by inhalation of Salbutamol (3-4 times a day)) all these above symptoms shows asthma is uncontrolled according to the classification of control of asthma. objective research data (in the lungs percussion-box sound, auscultatory - hard breathing, dry rales throughout the lungs
; laboratory methods data (in sputum:
); these instrumental methods of research (increased transparency of the pulmonary fields, enhanced pulmonary pattern, enlarged roots of the lungs). IT’s infectious dependent because of the history of pneumonia in the past
3. Make a plan for additional examination of the patient
* Study of the function of external respiration: spirometry (determination of the forced expiratory volume in 1 second - FEV1 and forced vital capacity of the lungs - FVC).
* Bronchodilator test (bronchial obstruction reversibility test)
* Allergic examination (skin tests, determination of specific IgE in blood serum, inhalation provocative tests with allergens)
* Chest X-ray (to exclude an alternative diagnosis).
*sputum eosinophilia
*Body plethysmography
4. Differential diagnosis
* Chronic obstructive pulmonary disease is characterized by long-term previous smoking or the presence of other risk factors, slow onset of respiratory symptoms, persistent or intermittent coughing during the day, progressive shortness of breath, the presence of irreversible bronchial obstruction, sputum eosinophilia is rarely present. Starts on average age
*chronic heart failure
*pulmonary embolism

  1. Treatment plan( identify the necessary groups of medications)
    * Diet:
    * Inhaled glucocorticosteroids (budesonide 200-400 mcg )in low doses in combination with long-acting beta2-agonists(salmeterol 50mcg 2 times a day thus 100mcg
    * Alternative: inhaled glucocorticosteroids at medium to high doses or inhaled glucocorticosteroids at low doses in combination with antileukotriene receptors.
    * Inhaled β-2 - rapid acting agonists when needed or a combination of inhaled glucocorticosteroids in low doses in combination with Formoterol.
17
Q

Task 60
A 47-year-old man was admitted to the emergency department with complaints of increased body temperature up to 38.5°С, cough with difficult to spit rusty sputum, pain in the right side of the thorax, aggravated by cough, the feeling of shortness of breath and dizziness.
In history: the patient fell ill 3 days ago, after hypothermia the body temperature increased and a cough appeared. The patient took medication on his own (Aspirin and Paracetamol), but the condition deteriorated, all the above symptoms appeared.
The ambulance took the patient to the hospital.
Objective clinical examination: the condition is severe. The skin is clean, cyanosis of the lips and fingertips. The right side of the thorax lags behind when breathing. There is a loss of resonance of percussion sound, increased bronchophony in the 4th and 5th intercostal space along the midclavicular line.
During auscultation, respiration is bronchial in the same part, with crepitation.
Cardiac sounds are muffled and quickened. Heart rate is 96 bpm. Arterial pressure is 85/50
mmHg. Saturation is 80%.
The abdomen is soft, no tenderness to palpation in all regions.
Clinical blood test was performed with the following results: WBC count 22×109/L, non-matured forms 10%, band neutrophils 23%, segmented neutrophils 30%, eosinophils 2%, lymphocytes 30%, monocytes 5%. C-reactive protein 125 mg/L.
Chest radiograph was performed in posteroanterior and lateral projections.

Task
1. Suppose the most likely diagnosis.
2. Justify the diagnosis.
3. Make and justify a plan for additional examination of the patient.
4. What is the treatment strategy for this patient? Identify the medications for the starting therapy
and the reserve medications?
5. What is the plan of dispensary observation of the patient after discharge from the hospital?

A

1.Diagnosis: Severe community-acquired right-sided middle-lobe pneumonia caused by an unspecified pathogen. Respiratory failure type 2.
2. Justify the diagnosis
* The diagnosis “community-acquired pneumonia (CAP)” was established on the basis of the patient’s complaints of an increase in body temperature up to 38.5 ° C, cough with hard-to-separate “rusty” sputum, pain in the right half of the chest, aggravated by coughing, feeling of lack of air, dizziness; anamnesis data (the patient fell ill outside the hospital) and an objective examination (dullness of the percussion sound, increased bronchophonia in 4-5 intercostal space along the midclavicular line on the right, with auscultation in the same department, breathing with a bronchial tint , crepitation). Localization was established according to the data of an objective examination and a chest X-ray (in frontal projection there is a darkening of the lower pulmonary field of the right lung; in the lateral projection, a lesion is seen mainly of the middle lobe of the right lung).
A severe course of community-acquired pneumonia was established on the basis of the presence of respiratory failure (cyanosis, dyspnea at rest, low oxygen saturation), vascular insufficiency (BP -85/50 mm Hg. Art.), hyperleukocytosis.
The degree of respiratory failure was determined by the level of saturation (80%).
3.make and justify a plan
* The patient is recommended to undergo MSCT of the chest organs to clarify the nature and volume of the lesion;
* monitoring of blood pressure and SpO2 for the timely transfer of the patient to the ICTiR
* biochemical blood test (urea, creatinine, electrolytes, liver enzymes, bilirubin, glucose, albumin) for early diagnosis of multiple organ failure;
* a detailed general blood test with the determination of the level of erythrocytes, hematocrit, leukocytes, platelets, leukocyte formula;
* bacterioscopy and bacteriological examination of a respiratory sample (sputum, tracheal aspirate, samples obtained during bronchoscopy) and blood with determination of the sensitivity of microorganisms to antibiotics;
*Fibrobronchoscopy
*spirography
4. What is the treatment strategy for the patient
*oxygen therapy 4-10 litters via medium concentration mask
*Iv fluids 0.9% solution to increase his blood pressure because it’s low and we measure the Bp after infusion
* In severe community-acquired pneumonia, antibiotic prescription should be urgent; a delay in starting antibiotic therapy for 4 hours or more significantly worsens the prognosis. The drugs of choice are III generation cephalosporins without antipseudomonal activity, Cefepim, inhibitor-protected aminopenicillins or Ertapenem, which should be prescribed in combination with an intravenous macrolide. For example AmoxicillinAmpicillin 0.5 or1g(preferably) orallyevery8hours
Reserve drugs: a combination of a respiratory fluoroquinolone (Moxifloxacin, Levofloxacin) with a third generation cephalosporin (Cefotaxime, Ceftriaxone), carbepenems. For example CA:-Cefotaxime-Ceftaroline-Ceftriaxone-Cefditoren 1.0-2.0gI/V,I/mevery6-8h RH:-Levofloxacin-MoxifloxacinGemifloxacin 0.5gevery12hor 0.75gevery24hinsideori/v
5.what is the plan of dispensary observation
* Dispensary observation is carried out within 6 months with visits to the local general practitioner 1, 3 and 6 months after discharge. A general blood test, sputum, a fluorogram of the chest organs, a spirogram are performed twice, after 1 and 6 months, a biochemical blood test - once every 6 months.
If necessary, consultations are carried out with a pulmonologist, an otolaryngologist, a dentist.
##Wellness measures: vitamin therapy, exercise therapy, sauna, sanitation of foci of infection, prevention of acute respiratory viral infections and influenza, smoking cessation, referral to specialized sanatoriums.

18
Q

Task 61
Patient M., 65 years old, went to the clinic with complaints of dyspnea during little physical activity, periodic slightly productive cough (especially during hypothermia in wet weather), a feeling of rale and chest tightness and palpitation.
In history: the patient has been smoking a pack of cigarettes a day for 30 years. Dyspnea appeared three years ago and began to increase over the past year. During the last month, after the patient suffered from acute respiratory disease, dyspnea and expectoration of sputum aggravated, the sputum became yellow and green. The patient has been observed for several years with the diagnosis of angina pectoris class II. Two years ago the patient suffered a large focal anterolateral myocardial infarction, and therefore the patient regularly takes Sotalolum, Cardiomagnyl, Monocinque (Isosorbide-5-Mononitrate).
Objective clinical examination: the condition of moderate severity. Mild acrocyanosis. Patient’s nutritional intake is low. The thorax is expanded in diameter. With percussion, there is a pulmonary box sound.
During auscultation of the lungs, respiration is diminished, with dry low-timbre and pneumosclerotic rale in the lower zones of the lungs. Exhalation is prolonged.
Respiration rate is 24 brpm. Heart rate is 100 bpm. Arterial pressure is 130/80 mmHg.
ECG was performed with the following results: scarring on the lateral wall of the left ventricle. Single ventricular extrasystoles.
Spirometry was performed with the following results: forced vital capacity 52%, forced expiratory volume in 1 s 37%, Tiffeneau-Pinelli index 57.2.
Chest radiograph revealed the signs of emphysema and pneumosclerosis of the lungs. General sputum analysis was performed with the following results: WBC per lpf, neutrophils 90%, lymphocytes 10%.
Task
1. State the provisional diagnosis.
2. Justify the diagnosis. Determine the smoking index of the patient.
3. Make a plan for further examination of the patient and justify it.
4. What basic therapy is necessary for the patient in accordance with his diagnosis?
5. What would you change in the basic ischemic heart disease therapy? Explain why..

A

1.Diagnosis: Exacerbation of emphysematous chronic obstructive pulmonary disease, GOLD 3, with fre- quent exacerbations, mMRC 3, group D. Concomitant disease: Arteriosclerotic heart disease. Angina pectoris II FC. Postinfarction cardiosclerosis (Q-forming of unspecified dura-
tion). Ventricular extrasystole. Chronic heart failure, stage I, Functional Class II.
2.Justify the diagnosis,determine the smoking index of the patient
* The diagnosis of chronic obstructive pulmonary disease (COPD) is made on the basis of complaints of progressive shortness of breath, cough, wheezing and heaviness in the chest, long smoking experience (30 years); objective data (emphysematous chest and percussion box sound, which, together with the predominance of shortness of breath over cough, indicates an emphysematous type of COPD). On the roentgenogram, changes characteristic of COPD are emphysema and pneumosclerosis. Using a spirogram, we clarify the diagnosis (obstruction) and stage - Tifno test <70% and FEV1 = 37% (III degree) This is why it’s Gold 3 due to airflow limitation. The presence of complaints of increased coughing, an increase and purulent character of sputum, confirmed by a general analysis of sputum, indicates the presence of an exacerbation. Anamnesis and ECG data indicate the presence of cardiac pathology. Smoker’s index - the number of cigarettes smoked per day × smoking experience / 20 = 20 × 30/20 = 30 pack / years. mMrc3( modified shortness of breath) Shortness of breath causes the patient to make stops when walking for a distance of about 100 meters or after a few minutes of walking on a flat surface group D
3. Make a plan for the further examination
*Body plethysmography
* Pulse oximetry to clarify the severity of respiratory failure, if possible - blood gases.
* General analysis of blood, urine. Blood glucose, CRP.
* To clarify the presence of an exacerbation of the disease, or concomitant diseases, or differential diagnosis.
* Microbiological examination of sputum to verify the pathogen
* ECHO-KS to exclude chronic pulmonary heart disease.
* Monitoring spirography after treatment to assess the reversibility of bronchial obstruction.
* Peak flowmetry daily to clarify the reversibility and variability of bronchial obstruction. If necessary - CT scan (presence of bullae in the lungs).
*bronchoscopy
4. What basic therapy is necessary
* Salbutamol inhaler (ventolin) 4mg 3 to 4 times a day
* Long-acting bronchodilators - primarily M-anticholinergic tiotropium bromide (Spiriva) 18 mcg (caps.) 1 time per day. It is possible to add a long-acting ß2-agonist - Formoterol (Foradil) 12 mcg (caps.) 2 times a day.
* On demand - inhalation of Berodual (ipratropium + fenoterol )through a metered-dose inhaler or nebulizer. With frequent exacerbations and a good response to glucocorticosteroids, the appointment of Ingal is indicated. GKS. An effective treatment option in such cases is the combination of ICS and a prolonged ß2-agonist, for example, Symbicort (Budesonide 160 mcg / formoterol 4.5 mcg). If shortness of breath persists, it is possible to add theophyllines per os (Teopek 2 times a day). With oxygen saturation <90% - long - term low-flow oxygen therapy. QC
5.what would you change in the basic ischemic heart disease
* The presence of serious competing pathology - ischemic heart disease dictates the need to take vital drugs. These drugs include the non-selective ß-blocker Sotalol , which has a side effect in the form of bronchospasm. Due to the presence of obstructive changes on the spirogram, it is recommended to replace it with a selective ß-blocker, for example, Bisoprolol 10mg

19
Q

Task 62
Patient B., 35 years old, was taken to the hospital by an ambulance with complaints of chest tightness, difficult breathing, especially exhalation, and a painful cough. The patient has had bronchial asthma for 10 years. Before that, the patient was observed with the diagnosis of chronic bronchitis for several years. During 5 years, the patient took 2 tablets of Prednisolone and had inhalations of Berotec for suffocation. Exacerbations of bronchial asthma occur 3-4 times a year and often require hospitalization.
The patient considers this deterioration of his condition to be associated with cancellation of Prednisolone a week ago. Negative allergic anamnesis. Attacks of suffocation are preceded by a short painful cough, which aggravates at the end of the attack. Then a small amount of viscous crudum sputum is produced.
Objective clinical examination: the condition of the patient is severe. The skin is pale, with a bluish tinge. The patient is sitting in the orthopnea position. The patient receives increased nutrition (gained 15 kg in weight in 3 years). The face is similar to Cushing’s face, there are striae on the thighs and abdomen.
The patient is excited and speaks using separated words. The thorax is in the position of deep inhalation. The abdominal muscles move during the act of breathing. Respiration is sharply diminished, with a small number of dry whistling rale, respiration rate is 32 brpm.
With percussion over the lungs, there is a box sound throughout all the lung fields, especially in the lower parts. Cardiac sounds are muffled and rhythmic. Pulse is 120 bpm, rhythmic. Arterial95 pressure is 140/90 mmHg. SpO2 is 85%.
The patient had more than 15 inhalations of Berotec( Fenoterol) in 24 hours.
The emergency doctor has already administered intravenously 10.0 ml 2.4% Euphyllin solution and Prednisolonum 60 mg.
Task
1. State the provisional diagnosis.
2. Justify the diagnosis. How can the deterioration of the patient’s condition be explained?
3. What should be done in this situation first?
4. What follow-up examination should be performed for the patient when the condition is stable?
5. Has the patient received the correct therapy in recent years? Why? What basic therapy should be prescribed to the patient?

A

1.Diagnosis: Exacerbation of severe persisting uncontrolled infection-dependent bronchial asthma. Status asthmaticus respiratory failure class II Complication: Exogenous hypercorticism.
2. Justify the diagnosis. How can the deterioration of the patient’s condition
* The diagnosis “bronchial asthma, endogenous form, severe persistent course” is made on the basis of complaints of frequent attacks of suffocation, frequent cough, feeling of tightness in the chest; anamnesis data (previous chronic bronchitis, the effect of treatment with corticosteroids and bronchodilators, frequent exacerbations of asthma requiring hospitalization). At the same time, the absence of allergic anamnesis
The patient developed a severe BA complication - asthmatic status of the 1st stage. (prolonged asthma attack , orthopnea, tachypnea, tachycardia, decreased blood oxygen saturation, weakened breathing in the lungs). SpO2 - 85% indicates ODN II Art. The presence of obesity, “cushingoid face”, striae on the body against the background of long-term intake of Prednisolone per os indicates exogenous hypercortisolism.
The deterioration of the patient’s condition is most likely associated with the complete cancellation of Prednisolone and the lack of basic therapy.
3.what should be done in this situation first
* Oxygen inhalation - 4 l / min. Inhalation of Berodual through a nebulizer (preferably O2) 3 times in 1 hour, after - 1 time per hour until the condition stabilizes
*Dexamethasone 2mg Intravenously once daily or
* IV corticosteroids (Prednisolone 60-120 mg or Hydrocortisone 200 mg). The daily dose of GCS IV in terms of Prednisolone (Metipred) is up to 600-700 mg .
* Drugs of the 2nd line - Euphyllin IV drip, adrenaline 1ml +NaCl
*IV, Magnesium sulfate IV.
* ICS (budesonide 160mcg)
Incase of Acidosis give Bicarbonate and magnesium sulphate
**
After stabilizing the patient we can collect blood samples; perform peakflow metry
4; what follow up examination should be performed for the patient when condition is stable
* Pulse oximetry to clarify the severity of respiratory failure (performed daily).
* General analysis of blood, urine. Blood glucose, CRP.
* General analysis of sputum, if possible, microbiological examination of sputum. Spirography. Peak flowmetry. Radiography of the OGK. ECG.
5. Has the patient received the correct therapy in the recent years
* In recent years, the patient received the wrong therapy: systemic corticosteroids should not be prescribed to BA patients as a basic therapy. The patient did not receive the drug at all from the main group of basic therapy - inhaled corticosteroids, as well as a prolonged bronchodilator. The presence of signs of exogenous hypercortisolism is explained by prolonged use of Prednisolone.
* The patient needs to be prescribed inhaled GCS + long-acting bronchodilators (ß2-agonists), better fixed drugs (Symbicort 160 / 4.5, 2 breaths 3 times a day or Seretide 25/500, 2 breaths 2 times a day). Considering the severity of asthma and the presence of chronic bronchitis, add M-anticholinergic long-acting Tiotropium bromide (Spiriva) 1 capsule (18 μg) per day to the treatment.
If shortness of breath persists, it is possible to add theophyllines (Teopek or Teotard). Against the background of basic therapy, a gradual decrease in the dose of Prednisolone is necessary, if possible until withdrawal or to the minimum dose at which there is no worsening of the condition.

20
Q

Task 43
A 52-year-old man was hospitalized from a doctor’s visit to the hospital with complaints of fever 39.5 °С, excessive sweating, shivering, dyspnea, dizziness during light physical activity, weakness, loss of appetite, and a 10-kg weight loss over the past 2 months.
The patient had been drinking alcohol for many years and smoking a pack of cigarettes a day for 30 years.
About two and a half months ago, the patient had his tooth extracted. A week after the procedure the patient experienced low-grade fever and excessive sweating. For two weeks, the patient took folk remedies, after which fever with shivering, excessive sweating, and pain in muscles and joints emerged. The patient did not seek medical care for another week. When the patient came to the lo- cal clinic, the chest radiograph did not show any pathology.
Blood test was performed with the following results: leukocytosis 14×109 / l and an increase in ESR to 24 mm /h . Ampicillinum (2 g a day) was prescribed to the patient. After getting the medication, the patient noted body temperature decrease to 37–38C and a decrease in join pain. Perspiration, severe weakness, and lack of appetite persisted. Five days after the end of taking antibiotics, the pa- tient again noted a rise in body temperature to 39°С with shivering.
Objective clinical examination findings: the condition was severe. The patient was inhibited. Or- thopnea. The body temperature was 39.5°С. The patient did not eat well. The skin was pale with yellow tinge, moist, with decreased turgor. On the palms, there were painless hemorrhagic spots 3 mm in diameter. There was no edema. Peripheral lymph nodes were not enlarged. With compara- tive percussion of the lungs, there was a box sound with loss of resonance in the lower parts. The borders of the lungs were normal. During auscultation, respiration was harsh, with moist rales in the lower zones of both lungs. Respiration rate was 24 brpm. Apex beat was in the sixth intercostal space along the left anterior axillary line. The borders of relative cardiac dullness were the follow- ing: the right one was 1 cm outwards from the right edge of the sternum, the upper was on the upper edge of the 3rd rib, the left one was along the left anterior axillary line in the 6th intercostal space. Auscultation of the heart was performed with the following results: rhythmic heart sounds, weaken- ing of the first sound in the 4th intercostal space along the left anterior axillary line, weakening of the second sound in the 2nd intercostal space to the right of the sternum. There was soft blowing protodiastolic murmur in the 2nd intercostal space to the right from the sternum and in the Botkin- Erb’s point and systolic murmur in the 4th intercostal space along the left anterior axillary line. Pulsus magnus was swift, rhythmic, symmetrical on both hands. Heart rate was 115 bpm. Blood pressure was 130 / 40 mm Hg, with carotid artery pulse.
The abdomen was soft and painless. The liver dimensions according to Kurlov were 14×12×10 cm. The liver was moderately tender to palpation. The spleen was soft and painless, palpated at the edge of the costal margin. The diameter of the spleen was 10 cm, the splenic length was 12 cm.
Complete blood count was performed with the following results: hemoglobin 85 g / l, WBC count 22×109/ l, band neutrophils 8%, segmented neutrophils 78%, eosinophils 1%, lymphocytes 8%, monocytes 5%. ESR 38 mm/h. Blood biochemistry showed the following: total bilirubin 41.2 mcmol / l, CRP +++, fibrinogen 6.5 g / l, albumins 40%, rheumatoid factor +.
Urinalysis was performed with the following results: specific gravity 1010, protein 0.99 g / l, 5–6 WBCs per high-power field (HPF), 5–8 lysed RBCs / HPF.
Echocardiography was performed with the following results: the thickness of the left ventricular
posterior wall and interventricular septum was 1.0 cm. Expansion of the left ventricular cavity: left ventricular end diastolic size was 6.0 cm, the size of the left atrium was 5.5 cm. There was an echogenic mass of 1.0×0.8 cm on the cusp of the aortic valve. Aortic regurgitation class III, mitral
73

regurgitation class II.
Task:
1. Determine the syndromes that the patient has.
2. Make and justify the clinical diagnosis.
3. What is the main additional examination that the patient has to undergo to specify the diagnosis? What is the correct way to perform this examination?
4. What are the main principles of treating this patient and indications for surgical treatment?
5. What measures should have been taken to prevent the development of this disease?

A
  1. Identify the syndromes
    * Syndrome of inflammatory changes and septicemia (fever, chills, acute phase indicators),
    * intoxication syndrome (weakness, excessive sweating, loss of appetite, weight loss, arthralgia, myalgia, anemia),
    *isolated urinary syndrome
    * valve changes syndrome (formation of aortic insufficiency, mitral insufficiency - mitralization of aortic insufficiency ), thromboembolic disorders syndrome (Jenway spots),
    * immune disorders syndrome (hepatitis, glomerulonephritis, splenomegaly, rheumatoid factor),
    * heart failure syndrome.
  2. Diagnosis: Primary diagnosis is primary aortic valve infective endocarditis. Aortic insufficiency, grade 3,Dilated left ventricle and left atrium, relative mitral insufficiency, grade II. Inflammatory response (hepatitis, glomerulonephritis). Anemia of moderate severity. Complications: CHF,
    stage IIA, FC IV.
    Justification of diagnosis: Rationale:
    great criteria (major)for echocardiogram IE - there is damage to the valves (vegetation by echo) and the pathogen is detected twice! (it is not, but it should be)
    Dukes criteria (Small criteria ): invasive interventions (tooth extraction), immunological phenomenon (spots, splenohepatomegaly, increased bilirubin, increased rheumatic factor, glomerulonephritis - mk hematuria and leukocyturia, anemia)
    Primary - there was no previous valve lesion
    Subacute - 2 months of illness and there were immunological phenomena, anemia . Activity 2-3 - high fever, increased acute phase proteins .
    3: what is the main additional examination
    * The patient is shown a bacteriological blood test to identify the causative agent of infective endocarditis and determine its sensitivity to antibiotics. Blood sampling for blood culture is carried out in compliance with the following rules. Should be taken at least three blood samples from different ve n. Blood collection from an indwelling catheter should be avoided due to possible contamination. Each time blood has been taken seeding in two vials for aerobic and anerobnoy flora . Each bottle contains at least 5-10 ml of venous blood.
    *chest x ray
    *ultrasound of Abdomen
    *lipid profile studies
    *coagulogram
  3. What are the main principles of treatment this patient and indications
    * The basic principles of treatment: early, massive and long-term (4-6 weeks) antibiotic therapy . Antibiotics with bactericidal action are used, the route of administration is parenteral. Prior to obtaining blood culture results, an empiric antibiotic regimen should be given, such gentamicin 3 mg/kg/day IV/IM divided q8hr for 4-6 weeks in combination with a beta-lactam and for 6 weeks when administered wiht vancomycin in combination with vancomycin (linezolid) or daptomycin.
    * Then ABT is prescribed in accordance with the results of the tank. blood tests. The main indications for surgical treatment: uncorrectable progressive heart failure; infectious process not controlled by antibiotics ; repeated episodes of thromboembolism; myocardial abscesses; endocarditis of the operated heart.
  4. What measures should have been taken to prevent the development of this disease
    ##High risk of developing infectious (bacterial) endocarditis in the following categories of patients :
    * Patients with mechanical prosthetic heart valves
    * Other types of prosthetics using prosthetic materials, including bioprosthetics
    * Postponed infective endocarditis
    * Congenital heart defects, such as: single ventricle of the heart, transposition of great vessels, tetrad of Fallot, including after surgical correction (so-called complex defects of the “blue” type)
    * Patients at risk of developing infective endocarditis should carefully monitor the condition of the oral cavity. Oral care is as follows:
    ● Brushing your teeth twice a day
    ● Flossing once a day
    ● Visit to the dentist every 6 months including professional cleaning of the oral cavity
    * Various medical interventions and the risk of developing infective endocarditis The following manipulations in the oral cavity requiring prophylactic antibiotics:
    ● Manipulations associated with violation of the integrity of the oral mucosa
    ● tooth extraction
    ● root canal treatment
    ● intraligamentary injection (this is an injection into the circular tooth ligament, when the injection is made through the periodontal sulcus)
    ● periodontal surgery
    ● other manipulations in which the periodontal joint is injured
21
Q

Task 46
A 38-year-old patient complained of thirst, decreased appetite, itching, and severe weakness. The amount of urine decreased to 300 ml per day( oliguria low urine out)normal urine output is 400-500 ml and above) about 2 cups of water)The patient vomited once upon admission. For 10 years, blood pressure had increased to 200 / 110 mm Hg.
Objective clinical examination: the skin was pale and dry, with subicteric tinge and traces of
75

scratching. There was no edema. The patient had vesicular breathing.
The heart sounds were rhythmic; the second sound was above the aorta(closure of aorta and pulmonic valves
The heart rate was 90 beats per minute. Blood pressure was 220 / 120 mm Hg. The abdomen was soft and painless. The liver and spleen were not palpable.
Complete blood count was performed with the following results: hemoglobin 72 g / l, WBC count 6.2×109 / l. WBC differential was within normal limits. ESR is 22 mm/h.
Urinalysis was performed with the following results: specific gravity 1.006, protein 0.66 g / l, 3–5 RBCs / HPF, 3–4 WBCs / HPF.
Creatinine in the blood serum was 1160 mcmol / l. Potassium in blood was 7.0 mmol / l(normal value 3.6-5.2mmol)
According to the ultrasound, the kidney dimensions were as follows: the right one was 8.0×3.5 cm, the left one was 8.2×3.8 cm, renal cortex was thin and outlined.(normal kidney dimensions 10-14cm in males length) 3.5 in width
Task:
1. State the most likely diagnosis.
2. What methods of examination are necessary in this case?
3. What treatment should be given to this patient?
4. What diseases should the differential diagnosis be made with?
5. What symptoms are mandatory for this disease?

A

1.Diagnosis: Arterial hypertension, stage 3 (systolic blood pressure > 180; diastolic blood pressure > 100). Hypertensive kidney disease. Chronic kidney disease, stage 5. Nephrotic syndrome Moderate anemia
2.what methods of examination are necessary in this case
* General analysis of blood, urine.
* Urine analysis according to Nechiporenko( to check numbers of RBCs ,white blood cells and cylinders or casts in urine. Zimnitsky(density of urine,it characterizes excretory abilities of the kidneys and renal concentrations
* Biochemical blood test: total protein, protein fractions, cholesterol, triglycerides, lipoproteins, potassium, sodium, seromucoid, fibrinogen, CRP
* blood creatinine , urea, glucose.
* X-ray examination of the lungs. ECG. ECHOKG. Examination of the fundus.
3. What treatment should be given to this patient
* Chronic hemodialysis. Kidney transplant.
4. What diseases should the differential diagnosis be made
* Chronic glomerulonephritis
* chronic pyelonephritis
* Renovascular hypertension
* Cohn’s syndrome
5.what are the symptoms are mandatory for this disease
* Severe weakness, nausea, itching, scratching. Pallor of the skin with a subicteric shade.
* Development of dry pleurisy, pericarditis is possible. In this case, the noise of friction of the pleura, pericardium is heard.
* Anemia in the blood. Oliguria. Decreased relative density of urine. In the biochemical analysis of blood, an increase in creatinine, urea, electrolytes.
Symptoms of uremia, symptoms associated with anemia,symptoms due to disorder of electrolytes,symptoms supported of hypertension

22
Q

Task 49
Female patient K., 70 years old, went to the physician for the first time.
Complaints: generalized weakness, shortness of breath upon little physical exertion, slight swelling in the regio cruris posterior, memory loss. Chest pain, no palpitations.
Past medical history: increased blood pressure for many years, the maximum is 180 / 110 mm Hg. The patient took medications irregularly, only when symptoms appeared. 15 years ago, type 2 dia- betes was diagnosed. The patient takes Metformin 1600 mg /day. The patient does not control gly-
cemia at home, does not follow a diet. 5 years ago, an ECG diagnosed cardiac arrhythmia. The pa- tient does not remember the names of prescribed drugs, which recently ran out. That is the reason why she went to the doctor. The patient complains of an increase in weakness and shortness of breath.
Objective examination findings: the condition is closer to satisfactory; increased food intake, height 165 cm, weight 90 kg. Respiratory rate – 20 breaths per minute. On percussion – the left border of the heart is 2 cm beyond the midclavicular line. On auscultation, there are few moist rales in the lower parts of the lungs, heart tones are significantly muffled, irregular rhythm, heart rate – 130 beats per minute, blood pressure – 165 /100 mmHg. Liver + 2 cm. Pitting edema of the calves. Results of additional tests: TSH – 2.1. Complete blood count – hemoglobin - 135 g / l, RBCs – 4.4x10 12, platelets – 210 thousand / l, ESR – 8 mm / h, WBCs – 4.5x10 9, eosinophils – 1%, band neutrophils – 0%, segmented neutrophils – 70%, lymphocytes – 22%, monocytes – 7%. Glycated hemoglobin – 9.3% and fasting glucose – 10.1 mmol / l. Lipid spectrum – total cholesterol – 6.0 mmol / l, LDL – 4.5 mmol / l, HDL – 0.9 mmol / l, triglycerides – 1.7 mmol / l. Serum creatinine – 93 mcmol / l.
Echocardiogram – hypertrophy of the left ventricle, ejection fraction – 38%.

Task:
1. Identify the main clinical syndromes, give a conclusion on paraclinical data.
2. Make a diagnosis.
3. Make a plan for additional examination
4. Use the CHA2DS2-VASc scale to assess the risk of thrombotic complications, determine whether there are indications for anticoagulant therapy.
5. Determine the goals of therapy, prescribe treatment.
6. Make a plan for follow-up.

A
  1. Identify clinical syndromes
    *heart insufficiency syndrome
    *metabolic syndrome
    *coronary insufficiency
    *hypertension syndrome
    1.Diagnosis: Arteriosclerotic heart disease: postinfarction cardiosclerosis, persistent atrial fibrillation, ta-
    chysystole 130 bpm. Chronic heart failure with reduced ejection fraction 38%, stage IIb, Functional Class III.
    Essential hypertension, stage 3 (systolic pressure 180 and more, diastolic pressure 110 and more), arterial hypertension 140-159/ 90-99. Left ventricular hypertrophy. Type 2 diabetes mellitus. Overweight. Dyslipidemia.
    ECG. Findings:irregular rhythm; S wave v2+R wave v5=35mm,Absence of P waves
    3:Make a plan for additional
    -Fundoscopy
    -coagulogram (PT;D-dimer)
    -coronary angiography
    -urinalysis
    -X-ray
    4:Use the indicator cHA2Ds2 vasc scale to assess the risk of thrombotic complications
    The score 6 points = stroke risk is 13.6% of stroke (TIA) systemic embolism
    -patient had coronary artery disease 1 point
    -Age 70 female(64-70years)is 2 points
    -heart failure with left ventricular hypertrophy 1point
    -Diabetes mellitus is 2 points
    ## chadsvasc >3 points in females is indicative for oral anticoagulation therapy
    Thus anticoagulant therapy is indicated
    5:Determine the goals of therapy ,prescribe treatment
    #goals of treatment is to prevent stroke in this patient and atrial fibrillation
    *Dabigatran 150mg or 110mg 2x day,apixaban(Eliquis)5mg twice a day starting with initial dose 2.5mg or 🛁rivaroxban 20mg orally one time a day
    *patient had Chronic heart failure Functional class III- 2 diuretics =Torsemide 10mg, spironlactone 25mg
    *Hypertension treatment= bisoprolol 10 mg
    *for diabetes- Metformin 1600mg/day
    *life style changes
    *Statins - Atorvastatin 10mg
    *Omega 3, salt restriction
    *calcium channel blockers = verapamil 240-480 divided into 3-4 equal doses a day
    *Amiodarone 300mg- 600mg for rhythm control
    6: follow up
    *Repeat ECG 24 holster monitor)
    *Bp monitoring
    *Glucose monitoring
    *Lipid profile
    *Assess symptoms improvement
    *Repeat ultrasound
    *coagulogram
23
Q

Task 50
A 65-year-old patient, a pensioner, went to the clinic with complaints of generalized weakness, fa- tigue, palpitation, dyspnea on exertion, nausea, eructation, and tightness in the epigastrium. There had been signs of gastric dyspepsia for about 15 years. In the last six months, the patient had begun to feel weakness, palpitation, dyspnea on exertion, and numbness of the lower extremities. The pa- tient did not go to the doctors.
During the examination, general condition was of moderate severity. The skin was pale with slight yellowish tinge of the skin and lemon-colored sclera. The patient’s face was puffy. Height 160 cm, weight 68 kg. There was vesicular respiration in the lungs. The borders of the heart were shifted to the left by 1 cm. The heart sounds were slightly muffled. Heart rate was 90 bpm. Blood pressure was 130 / 80 mm Hg. The tongue was crimson; the papillae were smoothed. The abdomen was soft and painless. The liver was palpated 1 cm below the costal margin, the spleen was not enlarged. Complete blood count was performed with the following results: hemoglobin 70 g / l, RBC count 2.9×1012 / l, CI 1.3, reticulocyte count 0.1%, WBC within normal limits, ESR 30 mm / h, MCV 70 fL. In blood smear, there were hypersegmented neutrophils(neutrophils whose nuclei have 6 or more than 3% of neutrophils with at least five lobes )Howell-Jolly bodies(remnants of DNA clusters in erythrocytes which is pathological and Cabot rings(Thread-like ring or figure eight shaped red blood cells ,likely remnants of mitotic spindles seen in defects of erythrocytes production especially in megaloblastic anemia)
Task:
1. What is the most likely diagnosis for this patient?
2. Justify the diagnosis.
3. Make a plan for additional examination.
4. What is treatment strategy?
5. Indicate the probable causes of this disease.

A

1.Diagnosis: Vitamin-B12-deficiency anemia, apparently associated with a deficiency of the intrinsic fac- tor.
2. Justify the diagnosis
* The clinical picture of B12 anemia is characterized by a triad:
1) blood damage: cyanocobalamin deficiency DNA fragmentation (blocking synthesis and disruption of cell division) megaloblastosis, accumulation of large forms of leukocytes and platelets, their early intraosseous destruction and shortening the life of circulating blood cells ineffective hematopoiesis, anemia, thrombocytopenia, leukopenia
In the patient: weakness, fatigue, palpitations, shortness of breath, hemoglobin and er are reduced.
As a result of increased destruction of erythra. - moderate jaundice due to indirect. bilirubin.
*2) damage to the gastrointestinal tract: glossitis - the tongue is crimson, the papillae are smoothed - “lacquered tongue”, hepatomegaly, heaviness in the epigastrium, signs of gastric dyspepsia approx. 15 years old, nausea, belching (decrease in B12 violation of the reproduction of epithelium cells to the gastrointestinal tract -> decrease in gastric secretion fermentation belching)
3) damage to the nervous system: cyanocobalamin is involved in the metabolism of myelin in the NS => with a deficiency of B12 - damage to the NS. It is manifested by funicular myelosis - the patient has numbness of the lower extremities (even mb ataxia, parasthesia, increased reflexes).
+ by OAC MCV 120 (increased), CP - 1.3 (increased) - hyperchromic, macrocytic anemia, hypersegmented neutrophils, Jolly and Kebot bodies, reticulocytopenia, increased ESR - characteristic of B12 anemia
+ puffiness of the face, which is also characteristic of B12 anemia (probably , but not precisely related to the fact that impaired DNA replication violation of protein synthesis osmotic pressure)
the severity is set according to Hb: for women 119-90 - mild, 89-70 - medium, below 70 - severe.
3. Make a plan for additional examination

  • myelogram: is the aspiration puncture of the bone marrow ,an exam that aims to verify the functioning of blood cells produced and megaloblastic changes,
  • Lactate dehydrogenase test :it is used to detect certain disease such as anemia: an increase in LDH, a decrease in alkaline phosphatase, an increase in indirect. bilirubin, serum iron, ferritin.
  • determination of vitamin B12 by radioimmunological method;
  • determination of the level of folic acid in the peripheral blood;
  • determination of methylmalonic and propionic acids, homocysteine in urine and / or blood.
  • FGDS, ultrasound OBP, irrigography - to establish the causes and identify the degree of gastrointestinal tract damage
  • ECG, Echo-KG - mb Long-term “experience” of pernicious anemia can lead to the development of myocardial dystrophy and heart failure. (note that the patient has a displacement of the boundaries of the heart by 1 cm, the tones are slightly muffled, there is shortness of breath, but it rather refers to the symptoms of anemia, and not cardiac)
    4. What is the treatment strategy
  • Intramuscular injections of vitamin B12 1000 mcg daily for 3-4 weeks (until the normalization of red blood counts).
  • Then maintenance therapy with vitamin B12 500 mcg 1 r / week for 2 months, then 500 mcg 1 r / month for a year, if not a removable cause, then 500 mcg 1 r / month for life.
    • etiological treatment (deworming / treatment of enteritis / stomach cancer, etc.
  • diet: liver, kidneys, eggs, milk.
  • if severe anemia / anemic coma develops - blood transfusions
    5. Indicate the probable causes of this disease
  • Alimentary insufficiency - insufficient intake of not only meat, but also dairy products and eggs.
  • Impaired absorption of B12 in the intestine: enteritis, chronic pancreatitis, celiac disease, Crohn’s disease, tumor of the small intestine, diverticula of the small intestine)
    • violation of the assimilation of cyanocobalamin as a result of deficiency of f-ra Kastl: atrophic gastritis, antibodies to f-ru Kastl or parietal cells of the stomach
  • Increased consumption of B12 - helminthiases
    ## Norms: Нb - 120-140, Er 3.7 - 4.7, CP 0.85 - 1.15, reticulocytes 0.5-1.2, tr 180-320, leuke 4-9, ESR 2-
    15, MCV 80-95. The norm of B12 in serum 100-250 pg / ml
24
Q

Task 51
A 28-year-old patient came to the hospital with complaints of fatigue, generalized weakness, epi- sodes of dizziness, palpitation, and dyspnea on exertion.
In history: the patient had noticed dry skin and brittleness of the nails for several years. Weakness and dizziness appeared a year ago during pregnancy. The patient’s menstrual cycle began at the age of 13. The menstruation was massive during the first 3–4 days. The menstruation was regular and lasted for 5–6 days. Currently, the patient was breastfeeding her child.
Objective clinical examination findings: the condition was of moderate severity. The skin was pale and dry, the nails were with transverse striation and exfoliate. The hair was brittle. Heart sounds were rhythmic. Heart rate was 90 bpm, blood pressure was 110 / 70 mm Hg. In the lungs, respira- tion was vesicular. The liver and spleen were not enlarged.
Complete blood count was performed with the following results: hemoglobin 75 g / l, RBC count 3.3×1012 / l, WBC differential was within normal values, ESR 12 mm / h, MCV 70 fL, MCH 21.0, anisocytosis(unequal sizes of red blood cells) poikilocytosis(an increase in abnormal red blood cells of any shape) .Serum iron was 6 mcmol / l.(normal is10.74 -30.4)
Task:
1. What is the most likely diagnosis for this patient?
2. Justify the diagnosis.
3. Make a plan for additional examination.
4. What is the treatment strategy?
5. What recommendations should be given to the patient for further disease prevention?

A

1.Diagnosis: Iron deficiency anemia, moderate severity.
2.Justify the diagnosis
* The diagnosis was made on the basis of the identified symptoms:
- anemic (fatigue, weakness, dizziness, palpitations, shortness of breath, pallor, decreased Hb, er)
- sideropenic (dry skin, brittleness, cross striation, foliation of nails, brittle hair.
Changes in the blood: Hb 75, er 3.3 (decreased), MCV 70 (decreased), MCH 21 (decreased), aniso-poikilocytosis, decreased serum iron.
3. Make a plan for additional examination
* IDA - microcytic, hypochromic.
* UAC (Hb, er, tr, leu, leukocyte. Formula, CP, hematocrit, MCV, MCH, MCHC, RDV, ESR) in dynamics to control therapy;
* BAC: a decrease in serum iron, ferritin, an increase in OOLSS, LVSS (latent serum iron-binding capacity).
* - additional laboratory and instrumental studies to identify the cause of the development of anemia: ultrasound OMT, OBP, Rg OGK, gastrointestinal tract, EGDS, consultation of a gynecologist
4. What is the treatment strategy
#Elimination of etiological factors;
* medical nutrition;
* treatment with iron-containing drugs:
orally, treatment for 2 months (relief of anemia) at a dose of 100-300 mg / day until the indicators normalize, and then maintenance therapy (saturation therapy - to replenish the depot) for 3 months in a waste dose 2-3 times less than with relief, 100 mg.
*ferrous sulphate 105mcg-500mcg + folic acid 800mcg tablet extended release :best taken on an empty stomach and patient should avoid dairy products,yea or coffee
vitamin C
* preparations containing ferrous iron (Ferrogradument, Sorbifer durules, Tardiferon, Totema, Aktiferrin, Ferretab, Hemofer)
-
preparations containing ferric iron (Maltofer, Ferrum lek, Maltofer foul, Ferlatum
* ferrous iron gluconate is better for oral administration
* Anti-reactive (supportive) therapy: taking iron supplements in short (7-10 days) courses monthly, at a dose of 100 mg / day. Consultation with a gynecologist and possible further treatment
* Additional: Evaluation of the effectiveness of IDA therapy: in the first days, subjective improvements, on days 5-8, reticulocytic crisis, from 3 weeks an increase in Hb and er, 4-6 weeks - the disappearance of hypochromia, normalization of Hb.
Norms: Hb 120-140, er 3.7-4.7 ESR 2-15, MCV 80-95, MCH 24-34, serum iron 10.7 -21.5)

  1. What recommendations should be given to the patient for further disease prevention
    *patient is recommended to take good nutrition rich with iron Good sources of iron include beans, dried fruits, eggs, lean red meat, salmon, iron-fortified breads and cereals, peas, tofu, and dark green leafy vegetables. Vitamin C-rich foods such as oranges, strawberries, and tomatoes help your body absorb iron.
25
Q

Task 52
A 38-year-old patient came to the district doctor with complaints of night-time epigastric pain, fa- tigue, dizziness, and palpitation on exertion.
In history: the patient had had peptic ulcer of the duodenum since the age of 17; the patient was twice hospitalized for peptic ulcer. During the last three months, the patient had noted episodes of health deterioration, weakness, dizziness, and dark color of the stool.
Objective clinical examination findings: the condition was relatively satisfactory. The skin and mu- cous membranes were pale. In the lungs, respiration was vesicular. Heart sounds were rhythmic. Heart rate was 80 bpm. Blood pressure was 110 / 70 mm Hg. The abdomen was soft and moderately tender in the epigastric region. The liver and spleen were not enlarged.
Complete blood count was performed with the following results: hemoglobin 73 g / l, RBC count 3.2×1012 / l, CI 0.71, reticulocytes 1%, WBC differential was within normal values, ESR 10 mm / h, MCV 70 fL, serum iron 7 mcmol / l.(normal range 10,7-30.)Total iron-binding capacity (TIBC) was 115 mcmol / l(normal range 42.96-80.55).(Fibrogastroduodenoscopy was performed with the following results: ulcer of the bulb of the duode- num 0.8 cm in diameter. Type F III.
Task:
1. What is the most likely diagnosis for the patient?
2. Justify the diagnosis.
3. Make a plan for additional examination.
4. What is the treatment strategy?
5. What are indications for bone marrow examination.

A

1.Diagnosis: Duodenal ulcer: an ulcer on the anterior wall of the duodenal bulb (d=0.8 cm). Chronic re-
current course, exacerbation. Recurrent bleeding. Type F III. Iron deficiency anemia (chronic posthemorrhagic), moderate severity.
2. Justify the diagnosis
* The diagnosis of duodenal ulcer was made on the basis of: anamnesis data from 17 years of age, suffering from duodenal ulcer, complaints of epigastric pain at night, which is characteristic of duodenal ulcer, objectively moderate pain in the epigastrium, FGDS data.
Aggravation because pain
GI bleeding - because anemic syndrome, dark colored stools.
IDA - on the basis of anemic s-ma: fatigue, dizziness, palpitations, pale skin and mucous membranes, Hb, er decreased, CP 0.71, MCV 70 - hypochromic, microcytic, serum iron decreased, TIBS increased.
Posthemorrhagic - as a result of bleeding from the gastrointestinal tract, chronic - within 3 months. Moderate severity: for men 129-90 - mild, 89-70 - medium, below 70 - severe.five.
3.Make a plan for additional examination
* OAC (Hb, er, tr, leu, leukocyte. Formula, CP, hematocrit, MCV, MCH, MCHC, RDV, ESR) in dynamics to control therapy
* BAC: a decrease in serum iron, ferritin, an increase in OOLSS, LVSS (latent serum iron-binding capacity).
* EGD (already available), Rg with contrast - examination of the gastrointestinal tract
* Detection of H. Pylori: PCR diagnostics - detection of H. Pylori DNA, urease breath test
4. What is the treatment strategy
## Elimination of etiological factors:
* eradication of H. Pylori: omeprazole / rabeprazole / esameprazole 20 mg 2 r / d + clarithromycin 500 mg 2 r / d + amoxicillin 1000 mg 2 r / d / metronidazole 500 mg 2 r / d + denol for 14 days
* treatment of anemia: exacerbation of ulcer duodenal ulcer is an indication for parenteral treatment, parenteral treatment is carried out with preparations of ferric iron (!): Maltofer, Ferrum lek, Maltofer fol. Ferlatum does not seem to have any form for parenteral administration.
calculation of the course dose for parenteral treatment according to the formula: A = M (Hb1-Hb2) 0.24 + D, where A - courses. dose, M - body weight, Hb1 - norm, Hb2 - patient hemoglobin, D - depot (500 mg in adults)
* In case they say that after all, per os: treatment for 2 months (relief of anemia) at a dose of 100-300 mg / day until the parameters normalize, and then supportive therapy (saturation therapy - to replenish the depot) for 3 months in the waste water the dose is 2-3 times less than when stopping, 100 mg.
- preparations containing ferrous iron (Ferrogradument, Sorbifer durules, Tardiferon, Totema, Aktiferrin, Ferretab, Hemofer)
- preparations containing ferric iron (Maltofer, Ferrum lek, Maltofer foul, Ferlatum)
- ferrous iron is better for oral administration
- therapeutic diet
5. What are indications for bone marrow examination
### When IDA is not done myelogram
It is possible for diff. diagnostics? Exclusion of other types of iron deficiency states - sideroblastic anemia, thalassemia.
In general, we talk about diseases in which a myelogram is done:
* All types of anemias (except for iron deficiency);
* tumor metastases;
* damage to one or more hematopoietic germs and the identification of atypical cells according to hemocytogram data;
-myelodysplastic syndromes;
-radiation sickness.
### Evaluation of the effectiveness of IDA therapy: in the first days, subjective improvements, on days 5-8, reticulocytic crisis, from 3 weeks an increase in Hb and er, 4-6 weeks - the disappearance of hypochromia, normalization of Hb.
Norms: hemoglobin 130-160, er 4.0 -5.1, CP 0.85-1.15, reticulocytes 0.5-1.2 ESR 1-10,
MCV 80-95, serum iron 14.0 - 30.4 TIBSS - 30-85

26
Q

Task 63
A female 47-year-old patient went to the hospital complaining of an increase in body temperature to 38 °C, chills, generalized weakness, dizziness, shortness of breath, palpitations, yellowing of the skin and sclera, pain in the lumbar region and left and right hypochondria, and darkening of urine. A week before, after hypothermia, the body temperature rose to 38 °C; the patient had had catarrhal symptoms (need to clear the throat,reduced sense of smell,chronic cough)for 3 days and took 500 mg paracetamol 2–3 times a day to manage them. The body temperature became normal, but there was yellowing of the skin and sclera and increasing weak- ness. There were episodes of jaundice in the past. The patient became acutely ill 3 days before, when the above complaints appeared within one day.
Objective examination findings: the condition is of moderate severity. The body temperature is 37.3 °C. The skin is pale yellowish with a lemon tinge, the sclera are icteric. The lymph nodes are not enlarged. Pulse - 100 per minute, regular, blood pressure – 95 / 60 mm Hg. The lungs: vesicular breathing without rales. The abdomen is soft, tender in the right and left hypochondria. The lower edge of the liver is felt 1 cm below the costal margin, moderately tender, the lower edge of the spleen is felt 3 cm below the costal margin on the left, tender on palpation. Daily diuresis is about 1 liter.
Results of laboratory tests:
Complete blood count: hemoglobin – 90 g / l, RBCs – 3.0×10 12 / l, color index – 0.85, MCV 100 fL, reticulocytes – 3%, WBCs 9.6x10 9 / l, WBC differential – lymphocytes 32%, segmented neu- trophils 57%, band neutrophils 4%, platelets 214x10 9/l, ESR – 18 mm / hour. Blood biochemistry: glucose 4.8 mmol / l, glycosylated hemoglobin 3.8%, total bilirubin 53.4 umol / l, indirect bilirubin 44.6 umol / l, AST 15 IU / l, ALT 17 IU / l, LDH 725 IU / l. Urinalysis: free hemoglobin was de- tected. Immunological test: positive direct Coombs test.(antiglobulins ; this means antibodies are acting against your RBCs and destroying them)
Task:
1. Identify the syndromes in this patient and determine the principle one.
2. Make and justify a diagnosis.
3. Make a differential diagnostic list and make a differential diagnosis for the primary disease.
4. What are you going to do next as a physician?
5. What treatment methods are used for this disease?

A

1.Identify the syndromes in this patient
*Anemic syndrome
*hemolysis
*splenomegaly syndrome:enlarged spleen,tender on palpation
*immune inflammatory syndrome
Main syndrome is Jaundice syndrome due to increased conjugated and unconjugated bilirubin,yellowing of skin and sclera
1.Diagnosis: Acute autoimmune hemolytic anemia, hemolytic crisis.
2. Justification : patient presents with symptoms of jaundice,anemia,and evidence of hemolysis.The positive direct coombs test indicates an autoimmune etiology,where the immune system is attacking and destroying red blood cells .The increased levels of indirect bilirubin and LDH further support the diagnosis of hemolysis
3: Differential diagnosis
*viral hepatitis
*Hemolytic transfusion reaction
*drug induced hemolysis
4:Next steps as physician
*confirm the diagnosis through further investigations,such as a coombs test,peripheral blood tests to assess for autoimmune markers and other possible causes
*Evaluate the extent and severity of hemolysis,anemia,and organ involvement through further imaging (e.g ultrasound and possible a bone marrow biopsy
*Assess the patient overall health,including comorbidities and potential triggers or underlying conditions contributing to the AIHA.
consult with a hematologist for further management and treatment planning
5:Treatment methods for autoimmune hemolytic
-corticosteroids:high dose or intravenous corticosteroids prednisone 5-7.5mg/day up to 20mg
-immunosuppressants:rituximab,cyclophosphamide or azathioprine
-splenectomy
-Blood transfusion
-support care : close monitoring of hemoglobin levels,organ function and potential complications,iron supplementation and folic acid maybe prescribed to support red blood cells production
**
Additional information

  1. The patient presents with symptoms of fever, chills, weakness, dizziness, shortness of breath, palpitations, yellowing of the skin and sclera, pain in the lumbar region and left and right hypochondria, and darkening of urine. The principle syndrome in this patient is jaundice with hepatosplenomegaly and anemia.
  2. Based on the symptoms and laboratory results, the most likely diagnosis for this patient is autoimmune hemolytic anemia (AIHA) with jaundice and hepatosplenomegaly.
  3. The differential diagnosis for AIHA includes viral hepatitis, drug-induced hemolytic anemia, and hereditary spherocytosis.
  4. As a physician, the next step would be to confirm the diagnosis of AIHA through further laboratory tests, such as a Coombs test and hemoglobin electrophoresis. A liver biopsy may also be considered to assess the extent of liver damage. Additionally, the underlying cause of AIHA should be investigated and treated accordingly.
  5. Treatment for AIHA may include corticosteroids, immunosuppressive therapy, and blood transfusions. In severe cases, splenectomy may be necessary. The underlying cause of AIHA should also be addressed and treated appropriately.
27
Q

Task 66
A district doctor of the local hospital called an ambulance with a view to possible hospitalization of a 45-year-old woman who was diagnosed with “Ischemic heart disease. Acute coronary syndrome. Bronchial asthma, exacerbation”.
In the history, the patient noted an increase in body weight over the past ten years after a significant decrease in physical activity. With body weight increase, the patient began to notice paroxysmal, mostly dry, painful cough accompanied by a feeling of shortness of breath.
After some time, the patient began to notice pain behind the sternum on exertion, which was most severe when working in the garden (weeding the vegetable patches).
The patient was examined on an outpatient basis. The diagnosis was “Severe bronchial asthma with a continuously relapsing course. Ischemic heart disease: exertional angina pectoris, class 3. Obesity, class 2”.
The prescribed medication included Prednisolonum, inhaled glucocorticoids, and bronchodilators and had little effect.
The patient said that nitrates were effective in relieving the chest pain within half an hour.
During the last 2–3 weeks, the patient noticed chest pain in the first half of the night, especially af- ter a late heavy dinner, which was the reason for seeking medical care.
The emergency ambulance team rejected the diagnose of acute coronary syndrome based on the clinical presentation, ECG results and high-sensitive troponin t testing.
During auscultation, respiration was harsh, without rales.
Chest X-ray revealed pneumofibrosis.
Task:
1. State the most likely diagnosis which combines all the patient’s symptoms.
2. What are the possible complications of this disease?
3. What special examination methods are used for this disease?
4. What recommendations should be given to the patient to change her lifestyle and diet?
5. What medications are used to treat this disease (specify the group and the name of the medica- tion)?

A

1.Diagnosis: Gastro-oesophageal reflux disease: reflux esophagitis, exacerbation. Hernia of the esopha-
geal orifice of the diaphragm
Justification of diagnosis: paroxysmal dry and painful cough accompanied by felling of shortness of breath ,patient notice pain behind the sternum on exertion and the diagnosis of bronchial asthma was misdiagnosed because prednisone and inhaled glucocorticoids and bronchodilators did not have any effect and even the diagnosis of acute coronary syndrome was misdiagnosed according to the ecg and troponin test and patient also noticed chest pain in the first half of the night especially after a late heavy dinner and the most definite sign in the chest x ray which reveals pneumofibrosis which could mean the reflux of acid got in to the lungs and caused pneumofibrosis
2.what are the possible complications of the disease
* Esophageal strictures
* bleeding from esophageal ulcers,
* Barrett’s esophagus,
* esophageal adenocarcinoma.
3. What special examination methods
* FEGDS
* histological examination
* pH-metric examination,
* manometry of the esophagus,
* X-ray examination,
* impedance measurement
* Test with Gaviscon for pain relief
4.Recommendations for patient to change lifestyle and diet
* Weight loss, smoking cessation, sleeping with an elevated head of the bed, avoiding medications that reduce the tone of the lower esophageal sphincter (theophyllines, nitrates, calcium channel inhibitors, antidepressants) that cause inflammation of the esophageal and gastric mucosa (NSAIDs, Doxycycline). Do not eat at night, do not take a horizontal position within 40-60 minutes after eating, avoid sour fruit juices, fats, chocolate, coffee, garlic, onions, peppers, alcohol, hot, cold and spicy foods, carbonated drinks.
5.what medications for treatment
* Proton pump inhibitors (Omeprazole 20mg 2-3 times a day Rabeprazole), prokinetics (Motilium10mg -30mg a day Ganaton 50mg ,antacids (Almagel 10-20mg, Maalox 175-200mg oral suspension)alginates (Gaviscon, Gaviscon forte).
***
Additional information

28
Q

Task 67
A 55-year-old patient went to the physician with complaints of pain of different intensity in the stomach (epigastric region), sometimes with irradiation in the back; pain occurred most often 1.5–2 hours after a heavy, spicy or fatty meal or alcohol intake and lasted up to 3 hours, intensified at night, in a supine position, decreased in a sitting position when the patient leaned forward and pulled the legs to the chest. Sometimes the pain irradiated to the left half of the chest. The patient also complained of significant nausea, lack of appetite, and bloating. After each meal for 1 hour, the patient had a loose, sometimes watery stool containing drops of fat. Weakness, a decrease in body weight, over the past year – by 7 kg, and intermittent pain in the calf muscles were reported.
Past medical history: the symptoms first appeared about 7 years ago, during the last 3 years there
were aggravations up to 4–5 times a year. The patient had experienced the current aggravation for
93

three days, after an episode of alcohol abuse.
Past history: the patient has been drinking alcohol daily for more than 10 years, mainly strong alco- holic drinks 200 ml per day on average, on weekends – up to 500 ml. The patient smokes 2 packs of cigarettes a day.
Objective examination findings: the condition is relatively satisfactory, the body height is 176 cm, the weight is 50 kg. Skin and mucous membranes are moderately pale, skin turgor is decreased. Pe- ripheral lymph nodes are not enlarged. Respiratory rate – 16 breaths per minute. Harsh breathing, few dry rales on both sides. Heart tones are muffled, regular, heart rate is 90 beats per minute, blood pressure is 140 / 90 mm Hg. On palpation of the abdomen: tenderness in the Chauffard’s zone, a positive Mayo–Robson’s sign(pain while pressing at the top of the angle lateral to rector spinal muscles and below the left 12th rib(left Costcovertebral angle, telangiectasia projected at the pancreas. Liver + 1 cm beyond the costal margin, the edge of the liver is elastic and tender on palpation, Lépine’s sign is negative. Cos- tovertebral angle (CVA) tenderness is absent on both sides. Completebloodcount:WBCs-7.8x109/l;RBCs–3.0x1012 /l;hemoglobin–98g/l;ESR–18 mm / h.
Blood biochemistry: α-amylase – 128 IU / l, lipase activity 90 IU / l; total bilirubin 22.0 mmol / l; GGT – 110 IU / l; AST 66 IU / l, ALT 35 IU / l; potassium – 3.3 mmol / l; total protein: 60 g / l; albumins – 30 g / l.(normal range 35-50g/l)
Amylase in urine – 480 IU / l.
Stool test (coprogram): steatorrhea++, neutral fat ++, fatty acid salts ++, creatorrhea ++. Pancreat- ic elastase 1 in feces – 120 μg / g.
Task:
1. Identify the main syndromes, make a conclusion on the results of laboratory tests, make the most likely preliminary diagnosis and justify it.
2. Make a plan for further examination. List the diagnostic signs of the disease according to the re- sults of medical imaging.
3. What are the main goals of conservative management/treatment of patients with this disease?
4. List the main drug and non-drug treatments that should be prescribed to this patient.
5. Make a plan for further management of the patient (follow-up) in the outpatient department.

A

1.Identify the syndromes and make a conclusion on the results of laboratory tests
-Exocrine pancreatic insufficiency (decreased elastase ,malabsorption,weight loss, stearrhoe
-pain syndrome
-Dyspeptic syndrome
-Anemic syndrome
*Diagnosis: Recurrent chronic toxic pancreatitis with combined clinical symptoms, with exocrine pan-
creatic insufficiency.
##TIGOR-O
T-toxic pancreatic
I-idiopathic
G-Genetic
A-Autoimmune
R-Recurrent (diabetes,cystic fibrosis)
O-obstructive
Alcohol ( men 40-45), obstructive 40 women
2.syndrome: severe malabsorption syndrome.
3. Research plan
*Electroyltes iron study,Na k
*Fecal elastic
*full blood test
*Glucose (glycated ,oral tolerance test, post pandril
* Determination of the level of amylase in blood and urine, scatological examination, determination of the level of elastase-1 in feces, X-ray of the abdomen, CT, MRI / MRCP, ERCP, ultrasound, endo-ultrasound
*coagulation profile
*Elastography of liver
4. Main goals of conservative treatment
* Stopping the use of alcohol, smoking.
* Determination of the cause of pain and an attempt to reduce its intensity
* Treatment of exocrine pancreatic insufficiency
* Identification and treatment of endocrine pancreatic insufficiency.
* Nutritional support.
* Screening for adenocarcinoma.
5.list main drugs and non drugs for treating this patient
* Refusal to drink alcohol. Diet low in fat, high in protein and carbohydrates 5-10 times a day NSAIDs (Paracetamol) 30 minutes before meals . If ineffective - Tramadol.
Enzyme replacement therapy with Lipase (20-45,000 for the main meal and 10-25 for the intermediate).
If symptoms persist, add proton pump inhibitors .
*creon 45000 lipase 40’000 iu
*patient should eat high calories,optimal fat foods

29
Q

Task 69
A 53-year-old male patient went to the hospital and complained of headaches, dry mouth, thirst (drinks up to 4 liters of fluid per day), generalized weakness, fatigue, nausea, vomiting, frequent urination, sometimes itchy skin, shortness of breath on exertion, heart problems.
Past medical history: the patient had been ill for 12 years, when edema of the face, shins and feet, and headaches first appeared. In subsequent years, he developed arterial hypertension (BP 160 / 100-190 / 110 mm Hg). Protein 1.2–2.4 g / l(protein range 1.2-2.4g/l is very severe proteinuria), leukocytes and erythrocytes were occasionally detect- ed in the urine. The patient refused renal biopsy. Despite the doctors’ recommendations, he did not receive elective regular treatment. He occasionally underwent treatment in the inpatient facilities, was prescribed prednisolone, ascorutin, and diuretics. The above treatment improved the condition: edema disappeared, protein in the urine decreased to 0.33 g / l. Recently, headaches had become persistent, the patient started to complain of nausea, vomiting, dry mouth, thirst, itching, and de- creased appetite; the patient lost weight.
Objective examination findings: the condition is of moderate severity. Reactions are a bit slow, the patient is apathetic and sleepy. The patient is undernourished, BMI 19 kg / m2. The skin is dry, pale, with traces of scratching. The face is puffy, there is soft pitting edema of the legs (a pit re- mains after pressing). The lungs: vesicular breathing, decreased breath sounds in the lower parts on both sides, without rales. The heart is enlarged – 2 cm to the left. Heart tones are muffled, frequent extrasystoles. Heart rate is 88 beats / min. BP 210 / 120 mm Hg. The liver and spleen are not en- larged. Costovertebral angle (CVA) tenderness is detected on both sides. Urination 8–10 times a day, including 1–2 times at night, urine is dark in color.
Additional tests:
Complete blood count: hemoglobin – 69 g / l, RBCs – 2.81012/ l, WBCs – 9.8109/ l, platelets – 126*109/ l, ESR – 75 mm / h.
Blood creatinine – 370 μmol / l. GFR – the formula SKD / EPI 14 ml / min / 1.73m 2.
Urinalysis: specific gravity 1003, protein – 0.8 g / l, 3–8 WBCs / HPF, 35 RBCs / HPF, hyaline casts 1–2, granular casts 1–2.
Daily albuminuria is 800 mg / day.
ECG – hypertrophy of the left ventricle, a decrease in voltage, prolongation of PQ, flat and two- phase T wave, frequent ventricular extrasystoles.
Ultrasound of the kidneys: right 8.1x3.7 cm, left 9.4 x 4.1 cm. The contours are uneven, the paren- chyma thickness is 14–15 mm, kidney tissue is diffusely hyperechogenic, blood supply is poor, cor- tical-medullary differentiation is impaired. There are few small cysts in both kidneys.
95

Task:
1. Name the main syndrome.
2. Prescribe additional tests (if necessary).
3. Make a diagnosis.
4. Prescribe the treatment

A
  1. Based on the provided information, here are the responses to your task:
    1 The main syndrome in this case is nephrotic syndrome, Chronic Renal Failure (CRF) or End-Stage Renal Disease (ESRD).
    2 Additional tests that may be necessary to confirm the diagnosis and assess the severity of the condition include:
    • Renal biopsy: This would provide a definitive diagnosis and information about the underlying cause of renal failure.
    • Serum electrolytes: To evaluate for electrolyte imbalances such as hyperkalemia or hyponatremia.
    • Renal ultrasound or CT scan: To assess the size, shape, and structure of the kidneys and to identify any structural abnormalities.
    • Kidney function tests: Including serum creatinine, blood urea nitrogen (BUN), and estimated glomerular filtration rate (eGFR) to assess the level of kidney function.
    • Urine protein electrophoresis: To determine the type of proteinuria and assess the severity.
    • Autoimmune markers: Such as antinuclear antibodies (ANA), anti-dsDNA antibodies, and complement levels to evaluate for autoimmune kidney diseases.
    3 The diagnosis based on the given information is Chronic Glomerulonephritis, which has progressed to end-stage renal disease.
    4 The treatment plan for this patient would involve a multidisciplinary approach and may include the following:
    • Referral to a nephrologist for further evaluation and management.
    • Blood pressure control: Antihypertensive medications (e.g., ACE inhibitors, angiotensin receptor blockers) to maintain blood pressure below 130/80 mm Hg, which helps slow the progression of renal disease.
    • Fluid and electrolyte management: Monitoring and correction of imbalances, such as avoiding excessive fluid intake and restricting sodium, potassium, and phosphorus as needed.
    • Renal replacement therapy: Considering the severity of renal failure, options such as hemodialysis, peritoneal dialysis, or kidney transplantation should be discussed with the patient.
    • Symptom management: Addressing specific symptoms such as headaches, dry mouth, thirst, itching, and shortness of breath through medications and supportive measures.
    • Nutritional support: Collaborating with a registered dietitian to develop a renal-friendly diet plan, emphasizing adequate protein intake and appropriate restriction of sodium, potassium, and phosphorus.
    • Monitoring and follow-up: Regular monitoring of kidney function, blood pressure, electrolytes, and other relevant parameters to assess the progression of the disease and adjust treatment accordingly.
    It’s important to note that the provided information is limited, and a comprehensive evaluation by a healthcare professional is necessary to confirm the diagnosis and determine the most appropriate treatment plan for the patient.
30
Q

Task 70
Patient N., 63 years old, is followed up by the district physician with the following diagnosis: primary disease: Cancer of the right kidney (T3N1M1). Right-sided nephrectomy performed in 2015. Chemotherapy and targeted therapy received in 2015. Disease progression in 2017 – metasta- ses in the lungs, bones, liver.
Underlying disease: Chronic pyelonephritis of the only left kidney. CKD, GFR 18 ml / min / 1.73 m2.
Complication of the underlying disease: chronic pain syndrome.
Examination of the patient during a visit to the physician.
Complaints. Nagging interscapular pain irradiating to the shoulder girdle with periodic episodes of shooting pain during angular movements and body rotation. The intensity of pain is 7–8 points ac- cording to the Numeric Rating Scale (NRS). Persistent pain in the right hypochondrium, aggravated after eating (pain intensity is 5–6 points according to the NRS). Cough, shortness of breath upon moderate exertion. A sharp decrease in appetite, weight loss, generalized weakness.
Anamnestic data. The patient has considered himself ill since 2015, when an ultrasound of the kidneys revealed a mass in the right kidney. Subsequently, the above-mentioned diagnosis was es- tablished, and special treatment was carried out. In 2017, the progression of the disease was detect- ed, the medical council of the cancer dispensary recommended symptomatic treatment at the district hospital.
Pain syndrome has been noted for a year, it gradually increases. For the past few months, the patient has been taking tramadol 100 mg 4 times a day, the drug showed a good effect. Then, within a week, the patient’s condition deteriorated. Intake of tramadol reduces pain by 20–30% for 1–2 hours. The patient often wakes up at night due to pain. He called the ambulance once due to in- creased pain, the ambulance team administered morphine with a good analgesic effect, while the patient noted excessive sleepiness, lethargy, and disorientation during the day after the injection.
Objective examination findings. The severity of patient’s condition is closer to moderate due to pain and generalized intoxication. The patient moves independently and slowly. Height – 178 cm, weight – 70 kg. Skin and mucous membranes are pale, skin turgor is moderately reduced. Respira- tory rate – 20 breaths per minute. Breathing is rough, with diffused dry rales. Heart sounds are muf- fled, rhythmic, heart rate is 96 beats per minute, blood pressure is 140 / 90 mm Hg. The abdomen is soft, with tenderness in the right hypochondrium. The edge of the liver is felt +5 cm below the cos- tal margin and is dense. No peripheral swelling is noted.
Task:
1. Formulate a comprehensive diagnostic opinion about the pain.
96

  1. Determine the stage of chronic kidney disease.
  2. Compile a treatment strategy for the pain syndrome. 4. Compile a certain drug therapy scheme.
  3. Name prescription forms for prescriptions.
A

1.Diagnosis: Severe chronic pain syndrome of complex origin (somatic and neuropathic), 7-8 points on
NRS.
2: 1 Diagnostic Opinion about the Pain:
2 The patient presents with nagging interscapular pain irradiating to the shoulder girdle, shooting pain during angular movements and body rotation, and persistent pain in the right hypochondrium aggravated after eating. The intensity of pain is 7-8 points according to the Numeric Rating Scale (NRS). These symptoms suggest a complex pain syndrome, likely resulting from multiple factors, including the underlying chronic pyelonephritis, cancer metastases, and previous nephrectomy. Further investigations are required to determine the specific causes contributing to the pain.
3 Stage of Chronic Kidney

4 The patient’s estimated glomerular filtration rate (GFR) is 18 ml/min/1.73 m², indicating a significantly reduced kidney function. Based on the Kidney Disease Improving Global Outcomes (KDIGO) classification, the patient’s CKD stage is Stage 4. It is important to note that the patient has only one functioning kidney due to the previous nephrectomy, which further complicates the management of CKD.
5 Treatment Strategy for the Pain Syndrome:
6 The treatment strategy for the pain syndrome should aim to address both the underlying causes and provide adequate pain relief. The strategy may include:
a) Multimodal Approach: Combining pharmacological and non-pharmacological interventions to manage pain effectively.
b) Targeted Therapy: Considering the use of targeted therapies specific to the underlying cancer and metastases, as guided by the patient’s previous treatment history.
c) Adjuvant Analgesics: Adding adjuvant analgesics such as gabapentin, pregabalin, or tricyclic antidepressants to enhance pain relief.
d) Palliative Care Consultation: Involving a palliative care specialist to provide comprehensive pain management and support for the patient’s overall well-being.
4 Drug Therapy Scheme:
5 The drug therapy scheme for pain management may include:
a) Opioids: Switching from tramadol to a stronger opioid such as morphine for better pain control. The dose and frequency should be adjusted based on the patient’s pain intensity and response, while closely monitoring for adverse effects.
b) Adjuvant Analgesics: Adding adjuvant analgesics like gabapentin or pregabalin to address neuropathic pain associated with cancer and its treatments.
c) Nonsteroidal Anti-inflammatory Drugs (NSAIDs): Considering the use of NSAIDs, taking into account the patient’s renal function and potential interactions with other medications. Caution should be exercised due to the patient’s compromised kidney function.
5 Prescription Forms for Prescriptions:
6 a) Morphine: Prescription for oral or sublingual immediate-release morphine tablets, e.g., 10 mg, 20 mg, or 30 mg tablets.
7 b) Gabapentin or Pregabalin: Prescription for oral capsules or tablets of gabapentin or pregabalin, e.g., 100 mg, 300 mg, or 600 mg capsules.
8 c) NSAIDs: If deemed appropriate and safe, prescription for oral NSAIDs such as ibuprofen, diclofenac, or naproxen, with recommended dosages based on the patient’s renal function and individual considerations.
It is important to note that the specific dosages and prescription details should be determined by the attending physician based on the patient’s individual characteristics, response to treatment, and any contraindications or interactions with other medications.
## Additional information
* the World Health Organization (WHO) has developed a pain ladder for the management of cancer pain. The pain ladder consists of three steps, with each step representing a different level of pain management.

Step 1: Mild to moderate pain.For mild to moderate pain, non-opioid analgesics such as acetaminophen (paracetamol) or nonsteroidal anti-inflammatory drugs (NSAIDs) such as ibuprofen or aspirin are recommended. These medications can be used alone or in combination with each other.

Step 2: Moderate to severe pain.If the pain is not adequately controlled with step 1 medications, then opioids such as codeine or tramadol are recommended. These medications can be used alone or in combination with step 1 medications.

Step 3: Severe pain.If the pain is still not adequately controlled with step 2 medications, then stronger opioids such as morphine or fentanyl are recommended. These medications can be used alone or in combination with step 1 and 2 medications.

It is important to note that adjuvant medications such as antidepressants, anticonvulsants, and corticosteroids may also be used to enhance pain relief and manage specific types of pain such as neuropathic pain or bone pain.

Here are some examples of medications that may be used at each step of the pain ladder:

Step 1:

•	Acetaminophen (paracetamol)
•	Ibuprofen
•	Aspirin

Step 2:

•	Codeine
•	Tramadol
•	Hydrocodone

Step 3:

•	Morphine
•	Fentanyl
•	Oxycodone

Again, it is important to note that prescription forms and dosages should be prescribed by a licensed healthcare provider based on the patient’s individual needs and medical history.